You are on page 1of 129

Test Information

Test Name ALL INDIA DAMS NEET PG 2022 CBT 4 Total Questions 210

Test Type Examination Difficulty Level Difficult

Total 210
840 Duration
Marks minutes

Test Question Language:-ENGLISH

1. An 85-year-old woman presents with muscle weakness, cramping, irritability and neuromuscular
excitability. Electrolytes reveal hypokalemia and a higher than normal plasma bicarbonate
concentration. Which of the following conditions will cause metabolic aklalosis?

a. Diarrhoea

b. Hypoaldosteronism

c. Renal failure

d. Treatment with a loop diuretic

Solution. Loop diuretics (such as furosemide inhibit Na-K-2Cl co-transporter in the loop of henle.
By inhibiting the transporter, loop diuretics reduce the reabsorption of NaCl and thus, water and
also increase the urinary loss of potassium. As a result, loop diuretics cause hypokalemia. Loop
diuretics also tend to cause alkalosis and hypokalemia by increasing distal tubule salt delivery,
which stimulates both K+ and H+ secretion.
Volume depletion produced by loop diuretics activates the RAAS. Angiotensin II increases
bicarbonate reabsorption in the PCT by increasing Na/H exchange, and aldosterone increases
distal bicarbonate reabsorption by promoting H+ secretion. Hypoaldosteronism will have the
opposite effect- that is, increase in plasma potassium and decrease in plasma bicarbonate level.
Diarrhea, renal failure produce metabolic acidosis with decrease in plasma bicarbonate levels.

Answer. d
2. An 18-year-old man presents with symptoms of vitamin B12 deficiency. Further diagnostic tests
reveal that he has pernicious anemia, the underlying problem in pernicious anemia is which of the
following?

a. Iron deficiency

b. Pancreatic insufficiency

c. Crohn disease

d. Autoimmune destruction of the parietal cells in the gastric mucosa

Solution. The cause of B12 deficiency in pernicious anemia is due to impaired intake of B12 due to
lack of intrinsic factor as a result of autoimmune destruction of the parietal cells in the gastric
mucosa.
Absorption of B12 occurs exclusively from the distal ileum, where specific receptors on ileal
enterocytes bind a complex of B12 and intrinsic factor. Although the intrinsic factor is secreted by
the gastric parietal cells, binding of the vitamin to IF occurs primarily in the proximal small
intestine. The acidic environment in the gastric lumen favours the binding of cobalmin to R
protein-type binding proteins that originate from salivary and gastric secretions. Pancreatic
proteases in the small intestine degrade the R proteins, and the rise in pH favours rapid and
complete transfer of the vitamin to intrinsic factor.

Answer. d
3. A 26-year-old woman has a premature delivery. Blood samples are taken from both the mother and
newborn infant for determination of their oxygen hemoglobin disassociation curves. If curve N in the
figure given below is the OHDC curve of the mother who has normal HbA, which of the curves is most
likely obtained from the premature infant?

a. A

b. B

c. C

d. D

Solution. HbF in fetal blood has a lower affinity for 2,3-BPG and therefore, a higher affinity for
oxygen. The fetal Hb OHDC will therefore, be to the left of the normal OHDC.

Answer. a

4. Which of the following stimulates the secretion of PTH?

a. Increase in extracellular calcium ion activity above the normal value

b. Increase in calcitonin concentration

c. Respiratory alkalosis

d. Increased secretion of PTH- releasing hormone from the hypothalamus

Solution. Respiratory alkalosis will decrease the ionic calcium in the plasma- this increases PTH
secretion.
Increase in ECF calcium would decrease PTH secretion. Calcitonin does not affect PTH secretion.
PTH-releasing hormone does not exist.

Answer. c
5. Which of the following is the most likely cause for renal sodium retention in compensated heart
failure?

a. Increased formation of angiotensin II

b. Increased release of atrial natriuretic factor

c. Sympathetic vasodilation of the afferent arterioles

d. Increased formation of ADH

Solution. In compensated heart failure, increased release of angiotensin II causes a direct


sodium retention and also stimulates aldosterone secretion that will, in turn, cause a further
increase in sodium retention by the kidneys. Because of the low arterial pressure that occurs in
compensated heart failure, the sympathetic output increases. This causes a sympathetic
vasoconstriction of the afferent arterioles of the kidney. This decreases the glomerular hydrostatic
pressure and therefore, the GFR, resulting in sodium and water retention in the body. The excess
sodium in the plasma will increase osmolality, which increases the release of ADH, causing renal
water retention (but not sodium retention.

Answer. a

6. Preganglionic fibres to the submandibular ganglion arise from:

a. Superior salivatory nucleus

b. Inferior salivatory nucleus

c. Nucleus of tractus solitarius

d. Nucleus ambiguous

Solution. • Superior salivatory nucleus sends the parasympathetic preganglionic secretomotor


fibres by facial nerve → chorda tympani nerve → submandibular ganglion. The post ganglionic
fibres are carried by the lingual nerve (branch of mandibular, Trigeminal.

Answer. a
7. Killian dehiscence is in:

a. Superior constrictor

b. Inferior constrictor

c. Middle constrictor

d. All of the above

Solution. • Inferior constrictor muscle has two parts: Upper thyropharyngeus made up of oblique
fibres and lower cricopharyngeus made up of transverse fibres.
• There is a potential gap posteriorly between the two parts called as pharyngeal dimple or
Killian’s dehiscence.

Answer. b

8. Cells which surround the oocyte in Graffian follicle are called?

a. Discus Proligerus

b. cumulus Oophorus

c. Luteal Cells

d. Villus Cells

Solution. Exp: Cells which surround the oocyte in Graffian follicle are called cumulus Oophorus
and site of attachment is called Discus Proligerus.

Answer. b
9. The following cell is inhibitory to

a. Vestibular nuclei

b. Deep nuclei of cerebellum

c. Anterior horn of spinal

d. Basal ganglia

Answer. b
10. Fibres from mammillary body ends in which marked area?

a. A

b. B

c. C

d. D

Answer. a
11. Ventilatory modes useful in weaning a patient from mechanical ventilation Include all except?

a. Synchronised intermittent mandatory ventilation (SIMV with pressure support

b. Bilevel ventilation (BIPAP with pressure support

c. Non-invasive ventilation (NIV

d. Airway pressure-release ventilation (APRV

Solution. Weaning from mechanical ventilation describes the transition between full
ventilatory support and spontaneous breathing, and the removal of any artificial airway.
Prolonged ventilation is problematic both for the patient (increased morbidity and mortality
and for the organisation (increased length and cost of stay.
SIMV is volume-controlled ventilation with or without pressure support for spontaneous
breaths.
It is not a suitable weaning mode when compared to others because of the mandatory breaths.
APRV is a ventilation mode that can be useful in patients with ARDS, where a high mean
airway pressure improves alveolar recruitment and oxygenation with carbon dioxide clearance
during the transient fall in airway pressure.
It is not suitable for weaning, because of the unnatural respiratory pattern.
The most common mode of ventilation for weaning is pressure-support ventilation.
Weaning may start with the patient on BIPAP, with reduction in ventilatory pressures,
cardiovascular support and sedation as the patient’s condition improves.
This facilitates a switch to spontaneous ventilation with gradually reducing pressure support as
the patient takes over the work of breathing.
T-piece trials may then also be used, although there is little evidence that they are superior to
simply reducing the pressure support the patient is receiving.
Continuous positive airway pressure (CPAP is the natural end-point of PSV weaning and may
be the last step prior to extubation.
Non-invasive ventilation is generally used in patients with chronic obstructive pulmonary
disease (COPD, where it may bridge the gap between ventilation and spontaneous breathing in
patients with poor respiratory reserve.

Answer. d
12. Hazards of O2 administration include

a. Retinopathy of prematurity

b. Bronchopulmonary dysplasia

c. Adsorption atelectasis

d. All of the above

Solution. Retinopathy of prematurity (retrolental fibroplasia is a hazard associated with O2


administration to neonates up to 44 weeks (gestational age + life age.
It is especially a hazard in the extremely premature (birth weight <1000 g and gestational age
<28 weeks.
Bronchopulmonary dysplasia is a chronic lung disorder that afflicts infants who required
mechanical ventilation at birth to treat respiratory distress syndrome.
CO2 retention is a hazard in patients with chronic obstructive lung disease.
Adsorption atelectasis is a potential hazard of oxygen administration in any patient receiving
oxygen concentrations greater than 50%.
It results from rapid uptake of oxygen into the circulation greater than the delivery of oxygen
by ventilation. Normally, the presence of nitrogen serves as an internal splint, protecting the
alveoli from collapse.
Prolonged high concentration of oxygen can damage “normal lungs” if given for prolonged
periods of time and may lead from mild irritation to tracheobronchitis to pulmonary interstitial
edema to pulmonary fibrosis

Answer. d
13. A patient on epidural anesthesia with 1.5% lignocaine develops hypotension and respiratory
depression in three minutes. Most common cause for this clinical condition is

a. Systemic toxicity of anesthetic agents

b. Drug allergy

c. Vaso vagal shock

d. Drug entering into subarachnoid space

Solution. EPIDURAL ANESTHESIA


• Drug is injected in the epidural space between the ligamentum flavum and the duramater.
• Tuohy needle - epidural needle
Indications for Epidural Anesthesia:
• Mainly used for controlling post - operative pain &Chronic cancer pain
• Painless labor
• Can be used for all surgeries done by spinal anesthesia
High levels of neural blockade can occur readily with either spinal or epidural anesthesia.
• Administration of an excessive dose, failure to reduce standard doses in selected patients (e.g.,
the elderly, pregnant, obese, or very short, or unusual sensitivity or spread of local anesthetic
may be responsible.
• Unconsciousness, apnea and hypotension resulting from high levels of spinal anesthesia are
referred to as a total spinal.
• One of the common mechanisms is inadvertent subarachnoid injection of local anesthetic during
and epidural block. Since a large amount of drug is used in epidural anesthesia so this drug in sub
arachnoid space can lead to very high levels of block that can cause medullary paralysis and
profound hypotension and apnea.

Answer. d
14. 1st step in the management of oculo cardiac reflex is?

a. Administration of atropine

b. Administration of glycopyrrolate

c. Release traction of extraocular muscles

d. Maintaining airway, breathing and circulation.

Solution. • Traction on the extraocular muscles or pressure on the globe causes bradycardia,
atrioventricular block, ventricular ectopy, or asystole. It is seen with traction on the medial rectus
muscle
• Stretch receptors are in the extraocular muscles.
• Once activated, stretch receptors will send afferent signals through the short- and long-ciliary
nerves. The ciliary nerves will merge with the ophthalmic division of the trigeminal nerve at the
ciliary ganglion. The trigeminal nerve will carry these impulses to the gasserian ganglion, thereby
resulting in increased parasympathetic tone and subsequent bradycardia.
• Prior Administration of an antimuscarinic drug such as glycopyrrolate or atropine reduces the
incidence of bradycardia during eye surgery
• In the event of arrhythmia, the anesthesiologist first should ask the surgeon to stop
manipulations. The ventilatory status is assessed. If significant bradycardia persists or recurs,
intravenous atropine is administered in 7-μg/kg increments.

Answer. c

15. The highest trace concentration of N2O allowed in the operating room (OR atmosphere by the
National Institute for Occupational Safety and Health (NIOSH is

a. 1 part per million (ppm

b. 5 ppm

c. 25 ppm

d. 50 ppm

Solution. • NIOSH sets guidelines and issues recommendations concerning the control of waste
anesthetic gases.
• NIOSH mandates that the highest trace concentration of N2O contamination of the OR
atmosphere should be less than 25 ppm.

Answer. c
16. Enzyme common to glycolysis and gluconeogenesis is/are which of the following

a. Phospho glycerate kinase

b. PEPCK

c. Pyruvate kinase

d. Phosphofructokinase

Solution. • Phosphoglycerate kinase is reversible enzyme which catalyzes conversion of 1,3


bisphosphoglycerate to 3 phosphoglycerate and vivc versa . It is used in glycolysis as well as
gluconeogenesis.
• Phosphoenol pyruvate carboxykinase (PEPCK is the enzyme needed for decarboxylation of OAA
to produce PEP and is used in the process of gluconeogenesis alone.
• Pyruvate kinase is the irreversible enzyme which converts phosphoenol pyruvate to pyruvate
and is used in glycolysis alone
• Phosphofructokinase is the irreversible enzyme which converts fructose 6 to fructose 1,6
bisphosphate and is used in glycolysis alone.

Answer. a

17. All are correct about telomerase except

a. It is an example of reverse transcriptase

b. It is found only in eukaryotes

c. It maintains high fidelity during DNA replication

d. It maintains the length of DNA

Solution. DNA polymerase enzyme is responsible for maintaining the fidelity of DNA while
replication is taking place. Telomerase enzyme is responsible for maintain length of daughter
DNA while replication takes place. Telomerase is a ribonucleoprotein having a RNA template in it.
RNA provides scaffolding on which telomere gets synthesized by reverse transcriptase activity.
Only eukaryotic DNA have this problem of end shortening as they are linear in nature, hence this
protection by telomerase and telomere is needed only in eukaryotic DNA and not in prokaryotic
DNA.

Answer. c
18. Which of the following are related to DNA Methylation
a Gene silencing
b Mismatch DNA repair
c RNA splicing
d Chromatin remodelling

a. a b d

b. a c d

c. a b c

d. b c d

Solution. DNA methylation play role in DNA silencing. Methylation of cytosine bases in CpG
island in gene promoter region is seen to repress the adjacent gene in embryonic DNA. It is
known as epigenetic phenomenon.
DNA methylation of parent strand at Adenine residue in GATC sequence results in hemi
methylated set of DNA. Methylated stand is then spared and nonmethylated strand is corrected
for any mismatching by mismatch repair system.
DNA methylation is involved in chromatin remodelling

Answer. a

19. Which all are gluconeogenic enzymes


a Pyruvate carboxylase
b PEP Carboxykinase
c Glucose 6 phosphatase
d Pyruvate kinase
e Hexokinase

a. a, b, c

b. b, d, e

c. a, c, e

d. c, d, e

e. Hexokinase

Solution. Pyruvate carboxylase and PEP Carboxykinase enzymes play role in conversion of
pyruvate to Phosphoenol pyruvate.
Glucose 6 phosphatase play role in conversion of glucose 6 phosphate to glucose in glycogenolysis
in liver and also in gluconeogenesis.
Pyruvate kinase and hexokinase catalyse irreversible reaction and are playing role in glycolysis
alone.

Answer. a
20. Protein is purified using ammonium sulphate by

a. Salting out

b. Ion exchange chromatography

c. Mass chromatography

d. Molecular size exclusion

Solution. • On adding the ammonium sulphate in a protein solution, protein undergoes


precipitation as ammonium sulphate attracts the shell of hydration to dissolve itself. This method
of protein precipitation is known as salting out.
• As a general rule higher the molecular weight of the protein lesser salt is required for
precipitation.
• Globulin is precipitated at half saturation and albumin is precipitated at full saturation.

Answer. a

21. A 24 year old man comes with feeling of pain and burning sensation on eating spicy food. He
reports some white lesions in oral cavity. The least likely diagnosis here would be:

a. Lichen planus

b. Candidiasis

c. Leukoplakia

d. Submucous fibrosis

Answer. c

22. Scalp scales are a feature of all except:

a. Discoid lupus

b. Cradle cap

c. Psoriasis

d. Alopecia areata

Answer. d
23. The following skin histology in a case of blistering disease is suggestive of:

a. Pemphigus vulgaris

b. Bullous pemphigoid

c. Linear IgA disease

d. Pemphigus foliaceous

Answer. a

24. Photo-distributed skin diseases are all except:

a. Discoid lupus

b. Airborne contact dermatitis

c. Lichen planus pigmentosus

d. Psoriasis vulgaris

Answer. d

25. Pick the incorrect pathology for these blisters:

a. Laminin: Epidermolysis bullosa simplex

b. BP-2: Cicatricial pemphigoid

c. Collagen: Epidermolysis bullosa dystrophica

d. Transglutaminase: Dermatitis herpetiformis

Answer. a
26. Match the medico-legally significant ages in table A with their medico-legal significance given in table B?

a. A-2,B-3,C-4,D-1

b. A-2,B-4,C-1,D-3

c. A-1,B-3,C-2,D-4

d. A-2,B-3,C-4,D-1

Solution.

Answer. b
27. The technique shown in the picture below, most commonly is associated with abuse of which
substance?

a. Methamphetamine

b. Grass

c. Crude opium

d. cocaine

Solution. Snorting is a technique where the abused substance is directly inhaled with a pipe
To nasal mucosa.
The absorption happens from nasal mucosa.
Cocaine, heroin are substances commonly abused by this technique.
In these people, upon long term abuse by snorting, nasal mucosa may show minute
Tears and nasal septum perforation also may be observed.

Answer. d

28. Ram is a prosecution witness. He turns hostile during evidencing. Public prosecutor can ask
leading question to him during?

a. Examination in chief

b. Cross examination

c. Can not ask leading question as he is prosecution witness

d. Re- direct examination

Solution. Leading question are allowed normally in cross examination conducted by opposition
Lawyer ( defense lawyer for prosecution witness If the prosecution witness turns hostile then
public prosecutor ( prosecution lawyer Can also ask leading question during Examination in Chief
( direct examination to Prove the hostility of his witness.

Answer. a
29. A drowned body is recovered from a well. Following will be noted during its post-mortem examination?

a. Hyponatremia & hyperkalemia

b. Hypernatremia & heavy and boggy lungs

c. Hyponatremia & heavy and boggy lungs

d. None of them

Solution.

Answer. d
30. Upon autopsy of a person who suffered a heavy blow on his head and died, following finding was
noted on head dissection. This finding is?

a. EDH

b. SDH

c. SAH

d. INTRACEREBRAL BLEED

Solution. The frequency of hemorrhages found at autopsy in head injury cases:


SAH> SDH> EDH
EDH is the least found in head injury.
But EDH is exclusive for head injury- it generates upon heavy and intense blow
To head only. Trivial head injuries do not lead to EDH.
95% of EDH are associated with skull fracture.

Answer. a
31. A 40-year-old man is brought to emergency department by his wife with complaints of dizziness
and blurring of vision for several hours. His wife adds that he has slurred speech since this morning
and was complaining of difficulty swallowing last night. His wife mentions that her husband was
working outdoors and eating stew with roasted beef and potatoes that had been sitting on the stove for
past 3 days. The past medical history is unremarkable. The physical examination reveals night eye
ptosis and palatal weakness with an impaired gag reflex. The cranial nerve examination reveals
findings suggestive of CN V and VII lesions. What is the mechanism of action of the toxin that most
likely is causing this patient's symptoms?

a. Expression of superantigen

b. Ribosylation of eukaryotic elongation factor-2

c. Inhibition of glycine and GABA

d. Inhibition of release of acetylcholine

Solution. This patient’s symptoms are consistent with botulism, which is caused by the
gram—positive, spore forming rod, Clostridium botulinum.
• Patients usually present with a flaccid type of paralysis, diplopial blurred vision, slurring of
speech, and weakness.
• Botulism is caused by a toxin released from C. botu (botulinum neurotoxin. which is a
metalloprotease that cleaves docking proteins (the SNARE proteins] and blocks exocytosis of
acetylcholine from storage vesicles at the neuromuscular junction, thereby producing flaccid
paralysis

Answer. d
32. A 36-year-old woman presents to the emergency department with a 2-day history of conjunctivitis,
sensitivity to bright light, and decreased visual acuity. She denies a history of ocular trauma. She wears
contact lenses and thought that the contact lenses may be the cause of the symptoms, although she has
always used proper hygiene. Fluorescein staining showed a corneal dendritic branching ulcer with
terminal bulbs that stained with rose Bengal. Giemsa staining shown in the image. What is the most
likely causative agent?

a. Human Herpes Virus 1

b. Varicella zoster virus

c. Acanthamoeba

d. Candida albicans

Solution. Dendritic ulcers are the most common presentation of HHV1 keratitis, which in turn Is
the most common cause of blindness and Indication for a corneal transplant in the developed
countries.
• HHV1 is a double-stranded DNA virus that is transmitted through direct contact with skin or
mucous membranes from an infected host.
• HSV keratitis initially presents as punctate keratitis and progresses to single or multiple linear
branching dendritic ulcers with terminal bulbs, swollen epithelial borders and central ulcerations
through the basement membrane.
• Giemsa staining shows multinucleated giant cells resulting from a coalescence of infected
epithelial cells and intranuclear viral inclusions.

Answer. a
33. Image shown here is a vaccine for COVI19. This vaccine belongs to

a. Sub unit vaccine

b. DNA vaccine

c. RNA Vaccine

d. Whole virus vaccine

Solution. COVOVAX is recombinant sub unit vaccine by using the Spike protein as an antigen.
• Recently approved from th GOI
• Dosage :0, 3 weeks

Answer. a
34. A 31-year-old man living in a remote tropical village presents with a swollen left leg and scrotum.
He says that his symptoms started over two years ago with some small multiple swellings near his
groin and have gradually, progressively worsened. He has also noticed that, over time, there has been
a progressive coarsening and fissuring of the skin overlying the swelling blood samples drawn at night
show worm—like organisms under microscopy as shown in the image. Which of the following
arthropods is the vector for the organism most likely responsible for this patient‘s condition?

a. Ades mosquito

b. Culex Mosquito

c. Tsetse fly

d. Sandfly

Solution. This patient is suffering from elephantiasis, a chronic form of lymphatic filariasis
caused by a microscopic, thread-like worm, Wuchereria bancrofti.
• The worms are transmitted to the healthy humans by the bites of infected Culex mosquitoes.
When mosquitoes bite an infected host, they take up microfilariae (immature larvae. These
microfilariae grow and convert into mature infective larvae within the mosquito.
• These infected mosquitoes will now bite healthy people and deposit the larvae on their skin from
where they enter the body. The larvae migrate to the lymphatic vessels in the body where they
develop into adult worms.
• Adult worms live for an average of 6-8 years within the lymphatic system causing chronic
inflammation, fibrosis, and blockage leading to edema and hugely swollen extremities.
• During its lifetime, the adult worm produces millions of microfilariae that circulate in the blood
and are taken up by incoming mosquitoes, thus maintaining an ongoing cycle of transmission.
• Elephantiasis is a form of chronic filariasis. It causes marked swelling of the lower half of the
body with coarsening and fissuring of the overlying thick skin. The arms, breasts, and upper body
can also be affected.

Answer. b
35. An 11-month-old boy is brought to the clinic by his mother for a rash on his trunk, which he has
had for the past 2 days. She notes that he is eating less and is more cranky than usual. His birth history
is insignificant, and his immunizations are up to date. Vital signs include temperature is 31.8°C, pulse
is 98/min, and respiratory rate is 16/min. The rash features thin-walled, fluid-filled blisters that rupture
easily. Fluid samples from the lesions are sent for analysis to a microbiology. The results reveal an
infection by the gram-positive bacterium Staphylococcus aureus. The patient is diagnosed with
staphylococcal scalded skin syndrome. Which of the following is involved with the primary defence in
response to the toxin produced by this organism?

a. Immunoglobulin- lgG

b. Immunoglobulin- IgE

c. Immunoglobulin- lgA

d. Immunoglobulin- IgM

Solution. Immune responses can be categorized as either primary or secondary. The primary
immune response is initiated by the production of lgM antibodies, and there is a significant lag
time.
• The secondary immune response results from a second exposure to the same antigen.
• The lag time is much shorter and the lgG antibodies are produced in significant quantities.
• These antibodies have a much higher affinity to the antigen than primary immune response
antibodies.

Answer. d
36. Ova of an image shown here is obtained from urine specimen of a NRI from Egypt. Choose the
correct Infective stage of this pathogen

a. Metacercariae

b. Pleurocercariae

c. Cercariae

d. Procercariae

Solution. Egg with terminal spine – S. hematobium


• Infective stage is Cercariae
• Causes Swimmer’s itch, Painless terminal hematuria.

Answer. c

37. Choose the correct pair

a. Stuart medium – Transport medium

b. PRAS medium – Anaerobic medium

c. Selenite F broth – Selective medium

d. Urease medium – Differential medium

Solution. Selenite F broth is an enrichment medium for enteric pathogens.


• Selective medium is solid in nature
• Enrichment meium is liquid in nature.

Answer. c
38. A 54—year-old gardener with diabetes mellitus acquired a small scratch from a thorn while
working in his flower garden. After 3 weeks, he noticed a small pink, painless bump at the site of a
scratch. He was not concerned by the bump; however, additional linearly distributed bumps that
resembled boils began to appear 1 week later that were quite painful. When the changes took on the
appearance of open sores that drained clear fluid without any evidence of healing as shown on the
image, he finally visited his physician. The physician referred to the gardener for a skin biopsy to
confirm his working diagnosis and to start treatment as soon as possible. Which of the following is the
most likely diagnosis for this patient?

a. Paracoccidioidomycosis

b. Blastomycosis

c. Cat scratch disease

d. Sporotrichosis

Solution. • Sporotrichosis (“rose gardener’s disease' is a rare fungal infection caused by


dimorphic fungi included within the Sporothrix species
• As these genera live in soil, on plant matter, rose bushes, and decaying material, people come in
contact with fungal spores present in the environment and thus acquire sporotrichosis.
• Although the fungus is present worldwide, endemically significant areas include China (Jullin
Province and the sub-Himalayan regions of India, and certain parts of Australia.
• The most common clinical presentation of sporotrichosis is a cutaneous infection on the hand or
arm, which occurs when the fungus is inoculated in the skin through a small scrape or a cut (most
often after handing contaminated plant matter.
• The condition is characterized by linearly distributed, painful ulcerated nodules that may
progress, ulcerate, and even become chronic in the form of verrucous plaques.
• Certain predisposing factors, such as diabetes mellitus. alcoholism or chronic obstructive
pulmonary disease, may contribute to the spread of the disease.

Answer. d
39. A 10-year-old boy is brought to the pediatric clinic with a complaint of a sore throat of 1-week
duration. He also has a cough with fevers. He adds that he has pain during swallowing and sometimes
water regurgitates from nose when drinking. He was diagnosed with acute tonsillitis by his primary
care physician 1month ago, for which he received a week-long course of amoxicillin. His immunization
status is unknown. On examination, he is alert and oriented to time, place, and person. 0n inspection of
his oral cavity, an oedematous tongue with a grey—white membrane on the soft palate and tonsils is
noted. The neck is diffusely swollen with bilateral tender cervical lymphadenopathy. Which of the
following organisms is the cause of this patient's condition and could have been prevented through
vaccinations in childhood?

a. Streptococcus pyogenes

b. Corynebacterium diphtheriae

c. Haemophilus influenzae b

d. Epstein Barr virus

Solution. Respiratory and cutaneous diphtheria are the main types of pathologies that
accompany C. diphtheriae infections. Humans are the only known reservoir of C. diphtheriae,
which is transmitted through respiratory droplets and direct contact with discharge from skin
lesions.
• C. diphtheriae has an insidious onset with an incubation period of 2—5 days.
• Initial symptoms of C. diphtheriae infections include a sore throat, difficulty, and pain during
swallowing. Malaise, and low-grade fever.
• An oropharyngeal examination shows a gray-white pseudo membrane over the tonsils. larynx,
and pharynx that bleeds on touch.
• Inflammation of the cervical lymph nodes and surrounding soft tissues gives rise to a 'bull-neck'
appearance

Answer. b
40. A 24—year-old married woman presents to the emergency department with a complaint of severe
abdominal pain since last night. She also complains of scanty vaginal bleeding. She says that she
visited a physician last year who said that she had a pelvic infection, but she was never treated because
of insurance issues. She also states that her period has been delayed this month. She is afebrile, the
pulse is 124/min, and the blood pressure is 100/70 mm Hg. On examination, the abdomen is distended
and tender. A pregnancy test was positive. A complication of infection with which of the following
organisms most likely led to this patient ‘s condition?

a. Candida albicans

b. Neisseria gonorrhoeae

c. Chlamydia trachomatis

d. Mycoplasma genitalium

Solution. This patient’s history of delayed menstruation features of severe abdominal pain. low
blood pressure, rapid heart rate, and distended and tender abdomen with a positive pregnancy
test are highly suggestive of a ruptured ectopic pregnancy.
• Given that she has a history of pelvic infection in the past. it is possible that the ruptured
ectopic pregnancy has occurred as a complication of the pelvic inflammatory disease. The most
common causative organisms of PID include Chlamydia trachomatis and Neisseria gonorrhoeae.
• Of the two pathogens, the most common cause is Chlamydia.
• The disease consists of chronic pelvic inflammation, which results in 3° scarring and fibrosis
• These patients have a high rate of ectopic pregnancy.
• This is one reason why pelvic inflammatory disease should be aggressively treated with
antibiotics.

Answer. c

41. Which of the following is absolute contraindication for copper T

a. Past history of PID

b. Past history of ectopic

c. Dysmenorrhea

d. Unexplained vaginal bleeding

Solution. Past history of PID, past history of ectopic pregnancy and dysmenorrhea are relative
contraindications for copper iud . while unexplained vaginal bleeding is absolute contraindication

Answer. d
42. Which of the following is safest drug for pregnancy induced hypertension

a. Captopril

b. Methyldopa

c. Hydralazine

d. Furosemide

Solution. Safest antihypertensive in pregnancy is methyl dopa. Hydralazine is given for acute
hypertensio in pregnancy. While captorpril or ACE inhibitors are contraindicated and diuretics
are not to be used as routine antihypertensive

Answer. b

43. a woman undergoing infertility treatment has the following image on usg, which of the following
drugs is likely cause

a. Injection GnRH agonist

b. Injection GnRH antagonist

c. Injection gonadotropin

d. Injection Hcg

Solution. This is an usg image of hyperstimulated ovary with multiple large follicles which will be
caused by injection gonadotropins, agonist will not cause such a large number of follicles.
Injection hcg is used as a trigger for ovulation

Answer. c
44. HPV 16 is associated with what percentage of cervical cancers

a. 30%

b. 50%

c. 70%

d. 90%

Solution. HPV 16 is responsible for 50% of all cervical cancers while HPV 16 and HPV 18
together cause 80% of all cervical cancers

Answer. b

45. pseudomeigs syndrome is associated with which of the following ovarian tumour

a. Fibroma

b. Granulosa cell

c. brenners

d. Mucinous

Solution. Meigs syndrome is seen with fibroma, granulosa cell, thecoma and brenners while
other tumours like mucinous when associated with ascites and pleural effusion is called
psuedomeigs

Answer. d
46. the following device is least likely to be used in

a. Removing iud

b. D&E

c. Suction evacuation

d. Endometrial biopsy

Solution. Uterine sound is used to measure length of uterine cavity as well as uterocervical
length, it is also used to confirm position of uterus before inserting any instrument and whenever
dilatation is required it is used as first dilator

Answer. a

47. Which of the following is an indication for surgical management of Ectopic Pregnancy?

a. Ectopic mass size > 4 cm

b. Serum beta hCG levels < 2000 miu/ml

c. Patient wants future pregnancy

d. Unruptured Ectopic Pregnancy

Solution. Indications for surgical management of ectopic pregnancy are


1 Unstable patient
2 Ruptured ectopic
3 Family complete
4 Contraindications to medical management
5 Failed medical management

Answer. a
48. A 24-year-old G2P1001 woman is at 34 weeks’ gestation and noted to be icteric. She also has nausea and
vomiting and malaise. A diagnosis of acute fatty liver of pregnancy is made, and the obstetrician recommends
immediate delivery. Which of the following is most consistent with acute fatty liver of pregnancy?

a. Elevated serum bile acid levels

b. Hypoglycemia requiring multiple D50 injections

c. Proteinuria of 500 mg over 24 hours

d. Oligohydramnios noted on ultrasound

Solution.

Answer. b
49. which of the following is not a characteristic finding on examination if the fetus is in the following
position

a. limbs in the flank

b. Infraumbilical flattening

c. Back towards flanks

d. FHR in the flank

Solution. The position is ROP


Diagnosis by abdominal examination
Non engaged head
Flat lower abdomen
Limbs easily felt in midline
Back towards flank
FHR towards flanks
By pelvic examination
High presenting part
Bulging membranes or early rupture of membranes
Easily felt anterior fontanelle towards pubic symphysis
Difficult to feel posterior fontanelle

Answer. a
50. which of the following is a component of AMTSL

a. early cord clamping

b. controlled cord traction

c. uterine massage

d. iv bolus oxytocin after delivery

Solution. WHO Recommendations for Active Management of the Third Stage of Labour (AMTSL,
2012 The use of uterotonics for the prevention of postpartum haemorrhage (PPH during the third
stage of labour is recommended for all births. Oxytocin (10 IU, IV/IM is the recommended
uterotonic drug for the prevention of PPH. In settings where skilled birth attendants are available,
controlled cord traction (CCT is recommended for vaginal births if the care provider and the
parturient woman regard a small reduction in blood loss and a small reduction in the duration of
the third stage of labour as important. In settings where skilled birth attendants are unavailable,
CCT is not recommended. Sustained uterine massage is not recommended as an intervention to
prevent PPH in women who have received prophylactic oxytocin. Postpartum abdominal uterine
tonus assessment for early identification of uterine atony is recommended for all women. CCT is
the recommended method for removal of the placenta in caesarean section

Answer. b

51. what is the treatment of choice for a 28 year old woman who on follow up for post molar evacuation
shows the following on chest Xray

a. Single dose methotrexate

b. Hysterectomy

c. EMACO

d. Multi dose methotrexate

Solution. Metastasis to lungs is considered low grade GTN and needs single agent
chemotherapy. The preferred agent is methotrexate. Alternating with folinic acid for 3 days in a
week, given for 3 weeks

Answer. d
52. match the following intrauterine infections with the characteristic clinical picture

Answer using the code given below

a. 1d 2b 3c 4a

b. 1b 2d 3a 4c

c. 1b 2c 3b 4a

d. 1b 2a 3c 4d

Solution. Parvovirus is trophic for erythroid cells and can cause fetal anaemia. Maternal infection
can lead to fetal hydrops, abortion, or stillbirth. In susceptible adults 20% to 30% will acquire
disease during school outbreaks
Microcephaly is important clinical finding in infants with congenital zika virus. In infants with
severe microcephaly show cutis gyrata or redundant scalp that is skin folds on scalp due to
continued growth of skin despite poor brain growth. These infants show hypertonia hyperreflexia
and spasticity.
Cytomegalovirus in the mother is usually asymptomatic, but 15% of adults will have a
mononucleosis-like syndrome. Maternal immunity does not prevent recurrence or congenital
infection. Congenital infection includes low birth weight, microcephaly, intracranial calcifications,
chorioretinitis, mental and motor retardation, sensorineural deficits, hepatosplenomegaly,
jaundice, anemia, and thrombocytopenic purpura

Answer. b

53. peripheral precocious puberty seen in all except

a. Hypothyroidism

b. Mc cune Albright syndrome

c. Hypothalamic hamartoma

d. Ovarian tumour

Solution. Although central precocious puberty is idiopathic in 90% cases some cases are caused
by CNS lesions including hypothalamic hamartoma, astrocytoma, ependymoma, optic glioma and
pinealoma. CNS irradiation can sometimes also cause precocious puberty. Peripheral precocious
puberty can be caused by functional follicular cysts, ovarian tumours like granulosa cell tumours,
Leydig cell tumours, gonadoblastoma and HCG secreting germ cell tumours.
Children with sever long standing hypothyroidism can present with [precocious puberty.
Exogenous sex steroids, adrenal tumours and McCune Albright syndrome are also causes of
peripheral precocious puberty

Answer. c
54. which of the following clinical findings is not seen in women with ashermans syndrome

a. Hypomenorrhea

b. Primary amenorrhea

c. Cyclic pain

d. Recurrent pregnancy loss

Solution. Recurrent pregnancy loss occur in ashermans due to abnormalities of implantation or


due to insufficient vascualrization. Cyclical pain occurs due to obstruction of menstrual flow and
/or hematometra. Infertility is reported in 7-40% patients. Abnormal uterine bleeding is seen in
70-95% of owmne with asherman syndrome. Most common is secondary amenorrhea followed by
hypomenorrhea.

Answer. b

55. all the following are seen with the following Mullerian anomaly except

a. Endometriosis

b. Cyclical abdominal pain

c. Transverse lie

d. Abortions

Solution. This is an image of unicornuate uterus which is least likely to be associated with
transverse lie as the space is less, it is more commonly associated with breech malpresentation. It
is a banana shaped uterus which is deviated to one side

Answer. c
56. a 45 year old woman presents with heavy irregular bleeding for last 6 months. Her pregnancy test
is negative and the following is the usg image obtained what is the diagnosis

a. Submucous fibroid

b. Endometrial polyp

c. Adenomyosis

d. Endometrial cancer

Solution. The usg shows the feeder vessel sign which is characteristic to endometrial polyp. The
treatment of choice is hysteroscopic polypectomy

Answer. b
57. which is not correctly paired

a. Endometrial biopsy for evaluation of infertility- day 26

b. Hysterosalpingogram- day 8

c. Gonadotropin evaluation- day 21

d. Postcoital test- day 14

Solution. The diagnostic evaluation of an infertile couple should be thorough and completed as
rapidly as possible. The primary diagnostic steps in the workup of the infertile couple include (1
documentation of ovulation by measurement of basal body temperature (BBT or mid–luteal phase
serum progesterone; (2 semen analysis; (3 postcoital test; (4 hysterosalpingogram; and (5
endometrial biopsy. Women should record their BBT for evidence of ovulation. In addition, serial
serum progesterone levels may be helpful to confirm ovulation. Serum progesterone values should
be obtained 7 days after ovulation and may also be helpful in evaluating inadequate luteal phase.
An endometrial biopsy may also provide valuable information regarding the status of the luteal
phase. The biopsy is obtained 12 days after the thermogenic shift, or 2 to 3 days before the
expected onset of menses, on about day 26 of a 28-day cycle. A postcoital test is an in vivo test
that evaluates the interaction of sperm and cervical mucus. It is performed during the
periovulatory period up to 12 h after coitus. The cervical mucus is obtained, and its quantity and
quality as well as its interaction with the sperm are evaluated. The hysterosalpingogram is
performed in the midfollicular phase in order to evaluate the fallopian tubes and the contour of
the uterine cavity; it should not be done while the patient is menstruating or after ovulation has
occurred. Although gonadotropin levels are not routinely evaluated, they should be obtained in
the early follicular phase when testing is indicated, e.g., in cases where there is a history of
oligoovulation.

Answer. c
58. what is the treatment of choice for the following Mullerian anomaly

a. Transcercical metroplasty

b. Tompkins procedure

c. Jones

d. Strassman metroplasty

Solution. This is most likely septate uterus and treatment of choice is hysteroscopic or
transcervical resecrtion of uterus or metroplasty and is done in the early follicular phase of the
menstrual cycle .

Answer. a
59. a Lady who is G6P0L0 with history of recurrent early pregnancy losses presents at 12 weeks with
missed abortion. Which of the following investigation is not warranted for her

a. VDRL

b. Fetal Karyotype

c. Lupus anticoagulant

d. Usg pelvis

Solution. We must start evaluating women after 2 failed clinical pregnancies including
biochemical pregnancies
Karyotyping of the parents and the abortus as it will help in picking up structural chromosomal
rearrangements usually a translocation or inversion.
Anatomic causes of pregnancy loss are typically diagnosed with sonohysterography, usg is useful
in pregnant women in whom other tests are relatively or absolutely contraindicated.
Sonohysterography provides more information than usg alone and is more accurate than hsg.
Hysteroscopy, laparoscopy and MRI can be performed if needed but are more expensive and more
invasive.
Anticardiolipin antibody and lupus anticoagulant should be done twice. Thyroid function test
should be done in women with clinical manifestations or a personal history of thyroid disease and
also in asymptomatic women as they have increased risk of miscarriage.
Routin evaluation for infections in women with RPL is not recommended in otherwise healthy
women.

Answer. a
60. what is the obstetric formula for a woman who is pregnant for the fifth time and has one twin
delivery at 34 weeks with both babies alive and healthy, 2 early spontaneous abortions and one
delivery of a IUD baby at 37 weeks

a. G5P1122

b. G6P1221

c. G5P2121

d. G6P1222

Solution.

Answer. a
61. All the statements are true about the device shown except

a. volume 50ml

b. can be used in out patient setting

c. generates pressure of 660mmhg

d. used up to 12 weeks

Solution. This is MVA syringe used for pregnancy termination by manual aspiration it generates
a pressure of 660mmhg. It is used as an alternative to suction evacuation and does not need
electricity and can be used upto 12 weeks and has a capacity of 60 ml.

Answer. a

62. For women undergoing induction of labor with a Bishop score of 5 or less, all of the following is are
used except

a. dinoprost

b. foley catheter

c. Cervical application of prostaglandin E 2

d. Laminaria tents

Solution. A Bishop score of 5 or less may lead to a failed induction as often as 50% of the time. In
these patients, prostaglandin E 2 (PGE 2 gel, PGE2 pessary (Cervidil, or PGE1M (misoprostol is
often used to “ripen” the cervix. Oxytocin is used to induce labor with a Bishop score greater than
5

Answer. c
63. what is the following instrument used for

a. Myomectomy

b. Hysteroscopic polypectomy

c. Tubal ring applicator

d. Hysterectomy clamps

Solution. A tubal ring, also popularly called a Falope Ring®, is a small silastic-ring shaped band
which is placed around the loop of the fallopian tube. During tubal ligation, a 2cm to 3cm segment
of the fallopian tube is drawn inside a narrow cone-shaped applicator. The silastic ring, previously
stretched around the applicator, is then released onto the tubal loop. Once the silastic ring
contracts, the fallopian tube is blocked. Deprived of blood supply, the constricted loop is replaced
with scar tissue and the remaining healthy tubal segments separate.
Contradictions
• Hemodynamic instability
• Uncorrected coagulopathy
• Severe cardiopulmonary disease
• Abdominal wall infection
• Multiple previous upper abdominal procedures

Answer. c
64. Match the letters A,B,C,D with the hormones they represent in the menstrual cycle

a. A-LH B-FSH C-ESTROGEN D-PROGESTERONE

b. A-FSH B-LH C- ESTROGEN D-PROGESTERONE

c. A-ESTROGEN B-PROGESTREONE C-FSH D-LH

d. A- PROGESTERONE B-ESTROGEN C-LH D-FSH

Solution. There is LH and FSH peak at the time of ovulation but the peak of Lh IS much bigger
than peak of fsh. Also estrogen is responsible for initiation of LH surge before ovulation Peak
levels of progesterone are seen 8 days post ovulation
LH surge happens 36 hrs before ovulation. While LH peak happens 12 hrs before ovulation

Answer. d

65. A 28-year-old G1P0 woman presents to the diabetes clinic at 28 weeks with a recent diagnosis of
GDM. By 37 weeks of gestation, she has been started on medical treatment with insulin before each
meal. The insulin dosing has increased until 36 weeks when her glycemic control was excellent with all
values below threshold. You schedule her for induction of labor at

a. 37 weeks

b. 38 weeks

c. 39 weeks

d. 40 weeks

Solution. Women with well-controlled pregestational diabetes and A2 GDM are usually induced
at 39 weeks of gestation. This allows enough time for the fetus to reach full maturity and
minimize fetal metabolic complications, but prevents stillbirth and overgrowth that would occur in
subsequent weeks of gestation. A1 GDM patients are usually managed expectantly until 40/41
weeks of gestation

Answer. c
66. A child was born at 34 weeks of gestation. Ophthalmologist review was sort to rule out retinopathy
of prematurity (ROP. On examination, the doctor confirmed that there are no changes of ROP, but he
could observe a glial tissue at the optic disc. Visual evoked response revealed normal response. What
can be the likely diagnosis among the following?

a. Optic nerve hypoplasia

b. Bergmister papilla

c. Papillitis

d. Congenital glaucoma

Solution. Bergmister papilla – asymptomatic condition. It’s a remnant of hyaloid vessel. VER is
normal.

Answer. b

67. A 40 year old female complains of blurring vision in both eyes, particularly in morning. She went to
see doctor who told her that it could be a corneal dystrophy. Which is more likely dystrophy out of
these to cause such symptoms?

a. Map-dot dystrophy

b. Macular dystrophy

c. Fuch’s dystrophy

d. Lattice dystrophy

Solution. Endothelial decompensation, like in Fuch dystrophy gradually leads to central stromal
oedema and blurred vision, worse in the morning.

Answer. c

68. First visual field defect in glaucoma is?

a. Isoptre contraction

b. Paracentral scotoma in Bjerrum area

c. Siedel scotoma in Bjerrum area

d. Bjerrum scotoma

Solution. A is 1st visual field change


B is 1st visual field defect

Answer. b
69. What is the correct diagnosis and amount of squint in the pic?

a. Right esotropia of 15 degree

b. Left exotropia of 30 degree

c. Left esotropia of 15 degree

d. Right esotropia of 30 degree

Answer. c
70. A diabetic female presents in eye OPD with diminution of vision. There are multiple white dots in
retina in all the quadrants in the given investigation. What is not true?

a. Dye is used in this investigation

b. Fluoroscence property is used in this investigation

c. Inner blood retinal barrier is breached in this picture

d. Stage is Proliferative Diabetic Retinopathy

Solution. Fluoroscein angiography


Diabetic retinopathy has inner blood retinal barrier break
Microaneurysms: These are localized outpouchings, mainly saccular, of the capillary wall that may
form either by focal dilatation of the capillary wall where pericytes are absent, or by fusion of two
arms of a capillary loop. Most develop in the inner capillary plexus (inner nuclear layer,
frequently adjacent to areas of capillary non-perfusion. This is the earliest sign

Answer. d
71. What is false for the given pic

a. Vertical axis is flatter

b. Patient has simple myopic astigmatism

c. The corneal curvature of horizontal axis would be more

d. -4DC at 180 degree will be the refractive spectacles given for this patient

Solution. Horizontal axis: more corneal curvature will have more steeper axis
Patient has simple myopic astigmatism
-4DC at 90 degree will be the refractive spectacles given for this patient

Answer. d
72. This test is done for?

a. Glaucoma

b. Squint

c. Colour vision

d. Vision testing

Solution. Hess charting being done


It aids in the diagnosis of ocular motility defects. It measures the deviation and the amount of
underaction and overaction of muscles

Answer. b
73. What should be the treatment of the condition in a 9 month old child of with the following condition

a. 2 lakh IU oral Vit A at 1,4, 16 days

b. 1 lakh IU oral Vit A at 0,1, 14 days

c. 1 lakh IU oral Vit A at 1,4, 16 days

d. 2 lakh IU oral Vit A at 0,1, 14 days

Solution.

Answer. b
74. A child with acute painful proptosis and small cells on histopathology which turned to be
synaptophysin positive. What is the diagnosis?

a. Rhambomyosarcoma

b. Leukemia

c. Retinoblastoma

d. Chloroma

Solution. Round cell tumour in a child with synaptophysin positive is in retinoblastoma.


Retinoblastoma can present as acute painful proptosis as well, though the most common
presentation is leukocoria

Answer. c

75. A patient has been on steroid inhalators for asthma for years. He notes blurred vision recently.
Which of the following findings are most likely related to the drug?

a. Anterior Subcapsular cataract

b. Posterior Subcapsular cataract

c. Corneal edema

d. Iris neovascularization

Solution. Long – term use of systemic corticosteroids often causes posterior subcapsular
cataracts and many cause glaucoma .

Answer. b

76. A 40 year old male got his routine blood workup done in which his ALP Levels came out to be 8-10
times the normal values. He told about the history of occasional backpain as well. Which of the
following statements ain't true about this condition?

a. This is likely to be associated with paramyxoviral infection

b. Salmon Calcitonin will be helpful in alleviating the bone pains

c. Bisphosphonates are the drug of choice

d. This is a low turnover bone disease

Solution. This is a clinical spectrum of Paget's disease. There is excess osteoblastic bone
formation and excess osteoclastic resorption leading to a high turnover bone disease.

Answer. d
77. A 76 year old lady sustains a slip and fall at home in washroom. She has been complaining of
severe pain and swelling in her hip and groin area. On thorough examination, its found that she has
shortening of approx.3.5 cm in her lower limb and lateral border of her foot is seen touching the bed
sheet. Which of the following treatment options best suits in this case ?

a. Internal fixation with cancellous and cannulated screws

b. Hemi Arthroplasty

c. Total hip replacement

d. Dynamic Hip Screw

Solution. Its an IT fracture Femur for which DHS/DCS/PFN is best suited

Answer. d

78. An 8 year old child sustains fall from height while flying kite on the roof. He develops gross
swelling over right ankle immediately. He is taken to a doctor whereby X rays were done which don't
show any bony fracture. The child is kept on conservative management.
Two years later, the child develops Calcaneovalgus deformity of the ankle. What did we miss Two years
ago ?

a. Salter Harris V Physeal Trauma

b. Avascular Necrosis of Talus

c. Secondary Osteoarthritis Ankle

d. Fracture Calcaneum

Solution. This is a classical case of Salter Harris VPhyseal Trauma where there is partial or
complete crushing injury to the physis. Since there is cartilaginous injury, its not usually evident
initially on X rays.

Answer. a
79. A 22 year old male fell down and sustained a twisting injury to left ankle. On examination there is a
lot of swelling around medial malleolus and X-rays In mortise view show increased medial joint space.
Which of the following structure is injured ?

a. ATFL

b. Spring Ligament

c. Deltoid Ligament

d. Talonavicular Ligament

Answer. c

80. The following X-ray shows lytic lesions of the lower limb. The biopsy shows fibroblastic
proliferation, osteoclast and inflammatory cells. From the given options, the most likely diagnosis that
can be inferred from the above clinical picture is:

a. LCH

b. Giant cell tumor (GCT

c. Chondroblastoma

d. Tenosynovial GCT

Answer. b
81. An expectant mother is seen for a prenatal visit. Prenatal genetic testing performed on her fetus
showed karyotype 45,X. She will be meeting with a genetic counsellor in the coming days.
Of the following, the MOST likely cardiac disorder to be seen in this fetus is

a. aortopulmonary window

b. coarctation of the aorta

c. tetralogy of Fallot

d. truncus arteriosus

Solution. A karyotype of 45,X, is consistent with Turner syndrome (TS. Patients with TS are
known to have an increased incidence of left-sided cardiac lesions consisting of bicuspid aortic
valve and coarctation of the aorta. Other vascular anomalies that have been noted in TS include
anomalous pulmonary venous connection and persistent left superior vena cava. Dilation of the
aorta in general, as well as dilation of other vessels such as the brachial and carotid arteries, may
also be seen. Tetralogy of Fallot, truncus arteriosus, and aortopulmonary window have not been
associated with TS. Affected patients can have resting sinus tachycardia and prolonged QT
syndrome

Answer. b

82. A 17 year old is seen for chronic nasal obstruction. She reports inability to breathe through her
nose for several months. She has a history of recurrent bronchitis, sinusitis, and chronic cough. Review
of systems is pertinent for bulky stools with a greasy film left in the toilet. The family history is
noncontributory. On physical examination, her weight is at the 80th percentile and body mass index is
at the 78th percentile. There is bilateral nasal obstruction with polypoid material filling both antra. Her
oropharynx is clear. Chest examination is notable only for a few inspiratory crackles in the left upper
chest posteriorly; no wheezing is heard. Point of maximum cardiac impulse is in the left midclavicular
line. Her abdomen is soft and there is no hepatosplenomegaly. There is no digital clubbing.
Serum IgE values are normal and radioallergosorbent test for multiple inhalant antigens shows
negative results. Pulmonary function testing shows a mild combined restrictive and obstructive pattern
with no bronchodilator response. Of the following, the MOST appropriate next diagnostic step is

a. CFTR mutation analysis

b. computed tomography of the sinuses

c. quantitative immunoglobulin determination

d. sweat chloride measurement

Solution. The presence of nasal polyposis, chronic cough with nonreversible obstruction on
pulmonary function testing, and signs of malabsorption make cystic fibrosis (CF the most likely
diagnosis for this adolescent. Although genetic testing for CF (CFTR mutation analysis is
indicated after the diagnosis is made, it is not the most appropriate diagnostic test. Measurement
of chloride in a sample of sweat obtained via pilocarpine iontophoresis remains the gold standard
for the diagnosis of CF.

Answer. d
83. A 6-month-old male infant who is limp and exhibiting obtundation in the morning is brought to the
emergency department. Bedside blood analysis reveals a significant metabolic acidosis, including an
elevated lactate level and a capillary blood glucose level of 35 mg/dL (1.9 mmol/L and 3+ ketones in
the urine. His newborn screening results were normal. His parents report that he only recently has
been able to sleep through the night without waking for a feeding and that he has had a few episodes
of morning lethargy that improved after his first morning feeding. The infant appears small and nearly
comatose. He has chubby cheeks, a short nose, and a relatively small chin. He has a temperature of
37.8°C, a heart rate of 155 beats/min, a respiratory rate of 15 breaths/min, and a blood pressure of
75/45 mm Hg. Cardiac examination reveals no murmur and a normal S1 and S2. The liver edge is
palpable 3 cm below the right costal margin. The remainder of the physical examination findings are
unremarkable. Of the following, the MOST likely diagnosis is

a. fatty acid oxidation disorder

b. glycogen storage disorder

c. mitochondrial disorder

d. organic acid disorder

Solution. The infant in the vignette, with obtundation after prolonged overnight fasting and
hypoglycemia with lactic acidosis and ketosis, most likely has a glycogen storage disorder (GSD.
Glycogen storage disease type 1a (also known as von Gierke disease is caused by a deficiency in
glucose-6-phosphatase. This type often presents in infancy and results in hepatomegaly, growth
failure, and recurrent episodes of hypoglycemia with ketosis. Although some neonates with
glycogen storage disease present with severe hypoglycemia, it is more common for infants aged 3
to 4 months to manifest hepatomegaly, lactic acidosis, hypoglycemic seizures, or a combination of
these. Additional laboratory abnormalities include elevated uric acid levels and hyperlipidemia.
The physical findings of chubby cheeks, a short nose, and a relatively small chin are nonspecific
but can be found in children with GSD. The diagnosis is made by means of genetic analysis
identifying pathologic variants in specific genes, including G6PC, SLC37A4, or both. These genes
encode for glucose-6-phosphatase activity and glucose-6-phosphate exchanger SLC37A4 activity,
respectively.

Answer. b
84. A 7-year-old boy who had surgery for craniopharyngioma 2 days ago now has increased thirst and
frequent urination. He has no vomiting, blurry vision, or seizures. His current medication includes
phenytoin. He has a heart rate of 102 beats/min, a respiratory rate of 16 breaths/min, and a blood
pressure of 102/60 mm Hg. He has no swelling over his body. His craniotomy incision site is clean and
healing. He has no neurological deficits, and the rest of his physical examination findings are
unremarkable.
Laboratory data are shown:
Sodium 150 mEq/L (150 mmol/L
Potassium 4.2 mEq/L (4.2 mmol/L
Chloride 110 mEq/L (110 mmol/L
Bicarbonate 23 mEq/L (23 mmol/L
Blood urea nitrogen 28 mg/dL (10.0 mmol/L
Creatinine 0.5 mg/dL (44 μmol/L
Glucose 90 mg/dL (5.0 mmol/L
Serum osmolality 325 mOsm/kg (325 mmol/kg
Urine Specific gravity 1.006, Leukocyte esterase, Negative, Nitrite-Negative, Blood-Negative, Protein-
Negative, Glucose- Negative
Of the following, the MOST likely diagnosis for this child is

a. acute kidney injury

b. central diabetes insipidus

c. cerebral salt wasting

d. syndrome of inappropriate secretion of antidiuretic hormone

Solution. The boy in this vignette is experiencing polydipsia and polyuria immediately after
undergoing cranial surgery. His laboratory evaluation shows dilute urine and hypernatremia,
favoring a diagnosis of central diabetes insipidus (CDI.

Answer. b

85. A 10 yrs old child with generalized edema has urine protein 3+ & serum cholesterol 238mg/dl. His
urine examination shows oval fat bodies
What is most likely diagnosis?

a. Nephrotic syndrome

b. Nephritic syndrome

c. Goodpasture syndrome

d. Urinary tract infection

Answer. a
86. In a 3 months old child with recurrent respiratory infections, which vaccine should not be given?

a. Measles

b. IPV

c. BCG

d. DPT

Solution. BCG should not be given to immunodeficiency patients

Answer. c

87. A 10 yrs old is brought to your clinic. Which vaccine should be given?

a. TT

b. DPT

c. Td

d. Others

Answer. c

88. A 9 years old boy has recurrent sino-pulmonary infections and greasy stools. Which of the following
is commonly seen in the boy?

a. Hypernatremia

b. Distal bowel obstruction

c. Protein losing enteropathy

d. Rectal prolapse is most common complication after treatment

Solution. Distal Intestinal Obstruction Syndrome

Answer. b
89. A child with coarse facial features, hypercalcemia and cardiac findings/murmur suggestive of
supravalvular aortic stenosis, likely defect is in?

a. 7p

b. 7q

c. 8p

d. 8q

Solution. William's syndrome; 7q11.23 deletion

Answer. b

90. In a fetus highest O2 concentration is found in?

a. SVC

b. IVC

c. Right Ventricle

d. Aorta

Solution. pO2 is highest in umbilical vein in fetal life followed by IVC

Answer. b

91. The direct cardiac effects of dobutamine would be blocked by which one of the following agents?

a. Prazosin

b. Metoprolol

c. Clonidine

d. Isoproterenol

Solution. Cardiac action of dopamine is due to activation of beta 1 receptor, that is blocked by
metoprolol

Answer. b
92. A 56-year-old man presents to his physician with persistent back pain due to an injury while
making a delivery 3 months ago. The patient said that he often feels persistent numbness and tingling
in his back, and the pain often travels down his leg. Which of the following medications may help
manage the patient's symptoms

a. Duloxetine

b. Fluoxetine

c. Phenelzine

d. Trazodone

Solution. Duloxetine is the SNRI useful for treating depression, fibromyalgia, neuropathy pain
and also stress incontenance

Answer. a

93. A 63-year-old man returns to his oncologist's office for a routine visit 2 months after undergoing a
partial colectomy for the treatment of colon cancer. He is currently on morphine for the management
of pain and is concerned about adverse effects. Which of the following effects would most likely remain
unchanged after starting morphine?

a. Euphoria

b. Miosis

c. Nausea and vomiting

d. Respiratory depression

Solution. All of the action of morphine may develop tolerance except miosis, constipation and
convulsion

Answer. b

94. Find out the true statements

a. Riociguat is the PGI2 analogue

b. PGE1 maintains the patency of ductus arteriosus

c. Piroxicam is the short acting NSAID

d. Lumaricoxib use is restricted because of its cardiotoxicity

Solution. Lumaricoxib use is restricted mainly because of its hepatotoxicity

Answer. b
95. A 24-year-old woman, who is 42 weeks pregnant, is admitted to the hospital for induction oflabor. A
fetal heart monitor shows that the fetus is in no acute distress. Sterile examination reveals that she is
minimally dilated without significant effacement. Due to an unfavorable cervix, she is given an agent
for cervical ripening prior to oxytocin administration. What medication was most likely given?

a. Carboprost

b. Dinoprostone

c. Epoprostenol

d. Treprostinil

Solution. Carboprost is the PGF2alpha agonist- useful to control post partum hemorrhage
Dinoprostone – is PGE2 analogue caused cervical riperning
Epoprostenol and Treprostinil are PGI2 analogue – useful for treating pulmonary hypertension

Answer. b

96. A 66-year-old woman with a long history of heavy smoking presents to her doctor with complaints
of shortness of breath and chronic coughing that has been present for about 2 years and has been
worsening in frequency. The doctor decides to prescribe a bronchodilator agent that has minimal
cardiac side effects, since the patient also has an extensive cardiac history. Which medication did the
doctor likely prescribe?

a. Albuterol

b. Theophylline

c. Ipratropium

d. Pseudoephedrine

Solution. Among these option irpratropium has least cardiotoxity

Answer. c

97. Regarding amiodarone which is the false statements

a. According to ACLS guidelines it is the preferred antiarrhythmic drug for treating cardiac arrest

b. Chronic therapy causes destruction of type II pneumocytes

c. It causes more hyperthyroidism than hypothyroidism

d. It causes reversible corneal microdeposits

Solution. Amiodarone cause more hypothyroidism than hyperthyroidism


Hypothyroidism is due to inhibition of peripheral conversion T4 to T3
Hyperthyroidism is due its iodine containing property and this drug also causes thyroiditis

Answer. c
98. Direct guanyl cycles activator useful in pulmonary hypertension is

a. Sivelestat

b. Bosentan

c. Riociguat

d. Selexipag

Solution. Riociguat is the direct guanyl cyclase activator


Bosentan is endothelin receptor blocker
Selexipag is the prostocycline receptor stimulant
Sivelestat is the extra cellular elastase inhibitor

Answer. c

99. All of the following are GABA analogues useful in treating epilepsy except

a. Tiagabine

b. Ezogabine

c. Stiripentol

d. Vigabatrin

Solution. Ezogabine is the potassium channel opener, useful for partial seizure

Answer. b

100. All of the following drugs are disease modifying drugs in CCF, except

a. Digoxin

b. Spironolactone

c. Valsartan

d. Carvedilol

Solution. Digoxin is the ionotropic agent, used only for symptomatic relief
Beta blocker, ACEIs, ARBs and spironolactone are used as disease modifying drugs in CCF

Answer. a
101. Find out the true and false statements
A. Sacubitril inhibits both vasopeptidase and angiotensin converting enzyme
B. Ranolazine aggravates movement disorder
C. Labetalol has norepinephrine reuptake inhibitor action
D. Treimethoprim causes hyperkalemia
E. Dofetilitde has least risk of causing QT prolongation

a. A-T, B-T, C.-T, D-F, E-T

b. A-T, B-T, C.-T, D-F, E-F

c. A-F, B-T, C.-T, D-T, E-F

d. A-T, B-T, C.-F, D-F, E-T

Solution. Sacubitril inhibits only vasopeptidase (neprelysin


Ranolazine aggravates movement disorder
Labetalol has norepinephrine reuptake inhibitor action
Treimethoprim has epithelial sodium channel blocking property so causes hyperkalemia
Dofetilitde – class III antiarrhythmic drugs causing more chance of QT prolongation

Answer. c

102. Find out the true and false statement regarding drugs used in covid 19 infection
A. Molnupiravir inhibits RNA-directed RNA polymerase
B. Molnupiravir Use in patients under 18 years of age can affect bone and cartilage growth
C. Molnupiravir administered by intravenous route
D. Remdesivir inhibits DNA-directed RNA polymerase
E. Remdesivir causes nephrotoxicity

a. A-T, B-T, C.-T, D-F, F-T

b. A-T, B-T, C.-F, D-F, F-F

c. A-F, B-F, C.-T, D- F, F-T

d. A-F, B-T, C.-F, D-F, E-F

Solution. Molnupiravir inhibits RNA-directed RNA polymerase


Molnupiravir Use in patients under 18 years of age can affect bone and cartilage growth
Molnupiravir administered by oral route
Remdesivir- recently approved for COVID19 , it is prodrug, given intravenously, acting by
inhibiting RNA dependant RNA polymerase, causing side effect of hepatotoxicity.

Answer. b
103. Mechanism of action drug

a. A.4, B.3, C.1, D.2

b. A.3, B.1, C.4, D.2

c. A.4, B.1, C.2, D.3

d. A.2, B.3, C.4, D.1

Solution. Alirocumab inhibits Proprotein convertase subtilisin/kexin9


Pravastatin inhibits hydroxy-3-methylglutaryl-coenzyme A reductase inhibitors
Niacin activates Lipoprotein lipase
Lomitapide is the Microsomal triglyceride transport inhibitor

Answer. a

104. Which agent is correctly paired with an appropriate clinical use of the drug?

a. Desmopressin—treatment of diabetes insipidus

b. Methyl ergometrine —induction of labour

c. Bromocriptine – Psychosis

d. Octreotide—treatment of infertility

Solution. Desmopressin—V2 analogue of vasopressin useful treatment of cranial diabetes


insipidus Methyl ergometrine —not useful for induction of labour, useful only for control of post
partum hemorrhage
Bromocriptine –is dopaminergic agonist causes adverse effect of Psychosis
Octreotide—somatostatin analogue has not role in the treatment of infertility

Answer. a
105. Match the following regarding antidiabetic drug and its site of

a. 1.B, 2.A, 3.D. 4.C

b. 1.A, 2.C, 3.D. 4.A

c. 1.D, 2.A, 3.B. 4.C

d. 1.C, 2.A, 3.D. 4. B

Solution. Pramlintide - Islet Amyloid Poly Peptide(amylin analogue, acting on hind brain, liver
and intestine , useful for both type 1 &2 DM
Pioglitazone- is the PPAR Gamma agonist, insulin sensitiser
Miglitol - alpha glucosidase inhibitor, useful to control post prandial hyperglycemi
empagliflozin is SGLT 2 inhibitor, acting on kidney (PCT, causing glycosuria

Answer. c

106. Find out the true statements


A. Lafutidine increases stomatostatin release
B. Pantoprazole competitively and reversibly blocking H K ATP ase pump
C. Lansoprazole need dose reduction in liver failure
D. Tenatoprazole short acting PPI

a. A, C

b. B, C

c. C, D

d. A, B

Solution. Lafutidine increases stomatostatin release. Lafutidine is H2 blocker, has additionally


nitric oxide releasing property, also increases mucous production.
Lafutidine decreases gastric hydrochloric acid secretion, improves ulcer healing and also has
ulcer protective activity.
Pantoprazole competitively and irreversibly blocking H K ATP ase pump
Lansoprazole need dose reduction in liver failure
Tenatoprazole long acting PPI

Answer. a
107. Which of the following is/are not the wrong statement?
A. Bedaquiline has cross resistance with clofazimine
B. Aztreonam has cross reactivity with ceftazidime
C. Thalidomide useful for type 1 lepra reaction
D. Voriconazole is effective for mucormycosis

a. A, C

b. A, B C

c. A, B

d. B.C.D

Solution. Bedaquiline has cross resistance with clofazimine


Aztreonam has cross reactivity with ceftazidime
Thalidomide useful for type 2 lepra reaction
Voriconazole is not effective for mucormycosis

Answer. c

108. Which one of the following is not useful in GIST

a. Imatinib

b. Regorafenib

c. Avapritinib

d. Ibrutinib

Solution. Ibrutinib is the bruton tyrosine kinase inhibitor useful in Mantle cell lymphoma, CLL

Answer. d
109. Match the following
Drug Mechanism of action
A. Rituximab 1. IL 1 inhibitor
B. Abatacept 2. IL 6 inhibitor
C. Rilonacept 3. B cell activation inhibitor
D. Sarilumab 4. T cell activation inhibitor

a. A-4, B-3, C-2, D-1

b. A-3, B-4, C-1, D-2

c. A-3, B-4, C-4, D-2

d. A-2, B- 4, C-1, D-3

Solution. Rituximab – targeting against CD 20 on B- lymphocytes and suppresses its function,


abatacept is fusion protein targeting against CD80/86 on T lymphocytes and inhibits T cell
activation, rilonacept is IL-1 blocker, sarilumab is the IL- 6 blocker

Answer. b

110. VGFR blocker approved for treatment of wet ARMD is

a. Ocriplasmin

b. Pegaptanib

c. Netarsudil

d. Sunitinib

Solution. vascular endothelial growth factor (VEGF antagonist useful in ARMD are
Intra vitreal injection
Bevacizumab, Ranibizumab
Pegaptanib –
Aflibercept –
Photodynamic therapy-given intravenous
verteporfin-
Sunitinib is tyrosine kinase inhibitor useful in RCC, GIST
Surafanib- useful in RCC, HCC
Axitinib useful in RCC

Answer. b
111. Which of the following is correctly matched regarding adulterants :

a. Ergotism: crotolaria seeds

b. Endemic ascites : Argemone oil

c. Epidemic dropsy : claviceps pupura

d. None of these

Solution.

Answer. d
112. A test for hepatitis C is performed for 400 patient with biopsy proven disease and 400 patient
known to be free of the disease . The test shows positive results on 200 of the patients with the disease
, and negative result on 360 of the patient without the disease . Which of the following is true
regarding this test :

a. Sensitivity is less than specificity

b. Sensitivity is more than specificity

c. Sensitivity is same as specificity

d. Data is insufficient to comment

Solution. Sensitivity = 200/400 = 50%


Specificity = 360/400 = 90%

Answer. a

113. All are true about p - value except :

a. Is the probability of committing Type - I error

b. Is equal t o 1- beta

c. Is the chance that the presence of difference is concluded when actually there is none

d. When p- value is less than alpha , the result is statistically significant

Solution. P value :
• Is the probability of committing Type - I error
• Probability of committing false positive error
• Is the chance that the presence of difference is concluded when actually there is none
• When p- value is less than alpha , the result is statistically significant
POWER : Is equal t o 1- beta

Answer. b
114. Which of the following is a primary prevention strategy?

a. Breast self examination

b. Physiotherapy

c. Radiation protection

d. MDT for leprosy

Solution. Primary Prevention-


a Actions taken prior to onset of disease which removes the possibility that a disease will ever
occur
b Intervention in pre – pathogenesis phase
c risk factors are present but disease has not yet taken place
d Modes of intervention
o Health promotion - Health Education, Environmental modification, Nutritional intervention
&lifestyle & behavioral changes
o Specific protection – Specific intervention. E.g. Immunization, chemoprophylaxis, use of specific
nutrient etc

Answer. c

115. Which tests are used in following situations :


1-Compare mean weights between 3 groups of obese patients
2-Mean blood sugar levels were compared before and after administering a new drug
3-Association between obesity and diabetes

a. 1-ANOVA , 2- paired t test , 3- Chi square test

b. 1-Student "t" test , 2- unpaired t test , 3- Chi square test

c. 1-Paired t test , 2- ANOVA , 3- Chi square test

d. 1-McNemar test 2- paired t test , 3- Chi square test

Solution. • Compare mean weights between 3 groups of obese patients: ANOVA test
• Mean blood sugar levels were compared before and after administering a new drug : Paired t
test
• Association between obesity and diabetes : Chi square test

Answer. a
116. Most important criteria to establish under Hill's criteria of causation :

a. Specificity of association

b. Sensitivity of association

c. Temporal association

d. Null association

Solution. • Temporal association :


o Suspected Cause should precede effect
o Most important criteria

Answer. c

117. Prevalence of a disease:

a. Is the best measure of disease frequency in etiological studies

b. Can only be determined by a cohort study

c. Is the number of new cases in a defined population

d. Describes the balance between incidence, mortality and recovery

Solution. Relationship between Prevalence, Incidence and Duration:


P=IxD
Larger the duration, greater the prevalence, shorter the duration (due to recovery or death, lower the
prevalence, compared to incidence rate.

Answer. d
118. A total of 50 patients with an atypical disease are identified and surveyed by patient interviews
with reference to their past exposure to radiation. This study most appropriately illustrates:

a. Case report

b. Case series

c. Case control study

d. Clinical trial

Solution. Case series :


• >1 Cases are studied with atypical manifestation
• No comparision group

Answer. b

119. Which of the following is/are true statements :


1: Under NIS - 1st dose of Pentavalent vaccine can be given till (Max limit : 2 year
2: Onset of dessiminated BCG infection after BCG is seen between (Time interval : 1 month to 12
months
3: Vaccine associated paralytic polio is seen after how many days of recieving OPV: between 4 to 30
days

a. 1,2,3

b. 1,2 only

c. 2,3 only

d. 1 only

Solution. • Under NIS - 1st dose of Pentavalent vaccine can be given till (Max limit : 1 year
• Onset of dessiminated BCG infection after BCG is seen between (Time interval : 1 month to 12
months
• Vaccine associated paralytic polio is seen after how many days of recieving OPV: between 4 to
30 days

Answer. c
120. Which of the following is/are true statements for the image shown below :

1: This is known as ogive curve


2: Median of this data is 30 cm
3: Interquartile range is : 10

a. 1,2,3
b. 1,2 only

c. 2,3 only

d. 1 only

Solution.

• This is known as ogive curve


• Used to plot cumulative frequency
• Median of this data is 30 cm
• Interquartile range is : 33-28 = 5

Answer. b

121. Which of the following is a true statement:

a. OPV is kept at the top of ILR

b. Hep B vaccine is not freeze sensitive

c. Subcentre is the last storage point of vaccine

d. OPV is not freeze sensitive

Solution. • OPV is kept at the floor of ILR


• Hep B vaccine is most freeze sensitive
• PHC is the last storage point of vaccine
• OPV is not freeze sensitive

Answer. d
122. A study revealed that in a study group, intake of betacarotene decreases carcinoma of colon but it
actually may be due to increased intake of dietary fibre.
This is due to:

a. Confounding factor

b. Misclassification bias

c. Randomization

d. Sampling error

Solution. Indirect association (Confounder : Statistical association between a variable of


interest and a disease due to another factor (confounding variable common to both the variable &
the disease.
Ex:
A study revealed that in a study group, intake of betacarotene decreases carcinoma of colon but it
actually may be due to increased intake of dietary fibre. Since those who had excess of
betacarotene had fibres along with them

Answer. a

123. Which of the following is a true statement :

a. Reservoir of plague : domestic rats

b. Scratches without oozing of blood is classified under Cat 3 exposure

c. Yersinia pestis is category A bioterrorism agent

d. Vector of scrub typhus : Soft tick

Solution. • Reservoir of plague : Wild rodents


• Scratches without oozing of blood is classified under Cat 2 exposure
• Yersinia pestis is category A bioterrorism agent
• Vector of scrub typhus : Trombiculid mite

Answer. c
124. SAANS scheme is to prevent deaths related to :

a. Pneumonia

b. Diarrhea

c. TB

d. Malaria

Solution. The Ministry of Health and Family Welfare has launched the campaign SAANS.
This scheme aims to reduce child mortality due to pneumonia and to mobilise people to protect
children from pneumonia, and train health personnel and other stakeholders to provide prioritised
treatment to control the disease

Answer. a
125. All of the following are disposed in red bag except :

a. Vaccutainers

b. Goggles

c. Syringes with fixed needles

d. Catheters

Solution.

Answer. c
126. A 35 year old women suffering from recurrent major depression becomes very distressed when
her supervisor asks her to revise a projects she has been working on for weeks. “I don’t do anything
right. I must be the most incompetent person in the firm. I will lose my job for sure” she state, sobbing,
to her therapist. According to the cognitive model, her depression is a consequence of which of the
following Problems?

a. Anger turned toward the self

b. Hopelessness

c. Worthlessness

d. Maladaptive negative beliefs

Solution. The lady here is having cognitive distortion (also known as maladaptive assumptions or
beliefs that she has “overgeneralised” her problem when she was asked to revise the project. She
did an arbitrary inference that I will lose my job for sure.

Answer. d

127. Which of the following is not true about waveform seen in EEG below?

a. Seen in NREM 2 sleep

b. Disorder in this stage is bruxism

c. Rapid eye movements

d. Reduced EMG levels in this stage

Solution. Sleep spindles and k complexes are seen in stage 2 nrem, with no rapid eye movements
and reduced EMG levels. Bruxism is associated with this stage.

Answer. c
128. All of the following are classified as gateway substances except?

a. Tobacco

b. Alcohol

c. Cannabis

d. Volatile solvents

Solution. Gateway substances are tobacco, alcohol and volatile solvents

Answer. c

129. Which of the following culture bound syndromes seen in india presents with acute anxiety and
panic like symptoms?

a. Koro syndrome

b. Dhat syndrome

c. Amok syndrome

d. Possession syndrome

Solution. Koro syndrome presents with acute anxiety about decreasing in size of genitalia.
Amok syndrome presents with violent, aggressive and homicidal behaviour
Dhat syndrome presents with getting weaker due to loss of semen.
Possession syndrome presents with multiple presentations including dramatizing behaviour.

Answer. a

130. In which of the following stages of Sigmund freud’s psychosexual stages of development, does the
male child develops a fear of getting castration?

a. Oral

b. Anal

c. Phallic

d. Genital

Solution. Phallic stage in males : oedipal complex (attraction towards mother and castration
anxiety (fear of castration
Phallic stage in females : electra complex and penis envy

Answer. c
131.

a. Pectoralis minor

b. Pulmonary artery

c. Pectoralis major

d. Subclavian vein

Solution. It is important to be able to identify the structures of the chest wall on all modalities,
including mammogram, ultrasound, CT, and MR.

Answer. a
132. An enlarged cardiac chamber is demarcated by the . To which chamber does it point? Choose
from the following:

a. Right atrium

b. Left ventricle

c. Left atrium

d. Right ventricle

Solution. The right atrium comprises the right heart border on a frontal radiograph.

Answer. a
133. What is the principal etiology of the mediastinal shift in this newborn with respiratory distress?

a. Atelectasis

b. Pneumothorax

c. Extralobar sequestration

d. Hypoplastic lung

Solution. Portable chest radiograph shows a massive right pneumothorax with shift of the
mediastinum to the left. A smaller left pneumothorax is also present. Note complete atelectasis of
the lungs. The principal etiology of the mediastinal shift in this case is the massive right
pneumothorax, which is under tension.

Answer. b
134. The radiograph shown below for better assessment of frontal sinuses. What is the common name
of the view shown.

a. Caldwell View

b. Waters View

c. Towne’s View

d. Pierre’s view

Solution. Please note the pterous is overlapping the maxillary sinuses and view has been done
for frontal sinuses mainly.

Answer. a
135. Identify the lesion shown on the CT scan

a. meningioma

b. glioblastoma

c. craniopharyngioma

d. oligodendroglioma

Solution. • Adamantinomatous craniopharyngiomas typically have a lobulated contour as a result


of usually being multiple cystic lesions. Solid components are present, but often form a relatively
minor part of the mass and enhance vividly on both CT and MRI. Overall, calcification is very
common, but this is only true of the adamantinomatous subtype (~90% are calcified

Answer. c
136. A 55 year old man comes to the physician with vertigo and disequilibrium while exercise. He says
that his symptoms are precipitated by exercising his arms or by raising his arms above his head. He
has coronary artery disease and hypertension and he is receiving treatment for that. He has bilateral
carotid bruits, rest of the physical exam is normal. What is the diagnosis of this patient ?

a. Peripheral artery disease

b. Cerebrovascular disease

c. Subclavian steal syndrome

d. Thromboembolism

Solution.

Answer. c

137. Which of the following according to consensus guideline is not an indication of surgery for
asymptomatic hyperparathyroidism?

a. Serum calcium level > 0.5 mg/ dl above normal. ( 1mg/dl

b. BMD by Dexa T score -2.5 at lumber spine, total hip, femoral neck or distal /3rd of radius.

c. Creatinine clearance < 60 ml/min

d. Age < 50 yrs

Answer. a
138. Which of the following is a false statement about this lesion?

a. This develops due to extravasation cyst from sublingual gland

b. It is very low malignant potential

c. Marsupialization is generally a good surgical option

d. Its plunging form may develop from submandibular gland

Answer. b

139. False stamen about salivary gland tumour is?

a. A most common site for minor salivary gland tumour is hard palate.

b. Adenoid cystic cancer has a tendency for peri-neural invasion

c. Mixed parotid tumour involving deep lobe should be treated by total conservative
parotidectomy

d. Best incision for parotidectomy is McFee incision.

Answer. d

140. Femoral Hernia is Nyhus classification is type?

a. Type 2

b. Type 3a

c. Type 3b

d. Type 3c

Answer. d
141. Pendelluft phenomenon is seen in?

a. Open Pneumothorax

b. Closed pneumothorax

c. Pension pneumothorax

d. Flail chest

Answer. d

142. False statement about SDH is?

a. Common in old age

b. Lucid interval is a classical feature

c. B/L and counter coup is more common than EDH

d. Bleeding is usually from Bridging veins

Answer. b

143. A untrained paramedical staff emptied water from the ICD bag (considering it a urobag and left if
dry. What will happen due to this action?

a. A closed pneumothorax becomes tension pneumothorax

b. A closed pneumothorax becomes open pneumothorax

c. The physician will not have any idea about its patency as he cannot see air bubbles

d. Detection of Bleeding in thorax will be delayed.

Answer. b

144. “LABEL” procedure is done for?

a. Gall bladder stones

b. CBD stones

c. pancreatic stones

d. Ureteric stones

Answer. b
145. Which of the following is not a boundary of Passaro triangle?

a. Ampulla of Vater

b. Junction of cystic duct with CBD

c. Junction of neck and body of pancreas

d. Junction of 2nd and 3rd part of duodenum

Answer. a

146. Which of the following node comes under D2 gastrectomy of for cancer?

a. Node along left gastric artery

b. Nodes along short gastric artery

c. Nodes along right gastro-epiploic.

d. Nodes along left gasroepiploic artery

Answer. a

147. False statement about hypertrophic scar?

a. Type I to Type III collagen ratio is 4:1

b. Does not extend beyond the scar

c. Does not grow beyond 6 months

d. No site predilection.

Answer. a

148. Which of the following is false statement regarding Sutures.

a. Subcuticular sutures have a better cosmetic outcome than intermittent suturing.

b. Skin stapling has better cosmetic outcome than skin suturing.

c. Cyano accrelate glue is an option for tissue approximation.

d. Steri Strips cannot be used for large wounds.

Answer. a
149. Screening MRI is recommended is all cases except?

a. BRCA mutation in first degree relative

b. H/O chest wall radiation in childhood for Hodgkin’s lymphoma.

c. Li. Fraumeni Syndrome

d. 10% risk in Gail assessment tool.

Answer. d

150. A 43 year old woman presented with serous discharge from a single duct of the nipple of her right
breast which was sent for evaluation. She is unlikely to be suffering from

a. Fibrocystic disease

b. Intraductal Papilloma

c. Duct Ectasia

d. Carcinoma

Answer. b

151. The Clavien-Dindo Classification is used for?

a. To assess pre-operative fitness of patient.

b. To asses fitness of cancer patient for chemotherapy

c. To assess recovery from trauma

d. To asses post operative complications

Answer. d

152. Which of the following statements regarding Ogilvie's syndrome correct?

a. It presents as acute large be mechanical obstruction

b. Marked caecal dilatation is a common feature on X-ray abdomen

c. Caecal perforation is a well recognized complication of this condition

d. Intravenous Neostigmine is for the treatment of this condition

Answer. a
153. Match the following pathological features of RCC?

a. A- 3, B-4, C-2, D-1

b. A-2, B-4, C-1, D-3

c. A- 4, B -3 C- 1, D -2

d. A- 3, B -1, C- 4, D- 2

Answer. a

154. False statement about pheocromocytoma is?

a. Paroxysmal presentation is the most common sign

b. Found with Wermer’s syndrome

c. In preoperative preparation always first alpha blocker should be gives

d. MRI gives light bulb sign

Answer. b
155. A patient was kept on anti-coagulant due to recurrent DVT. He was planned for BPH surgery
(AUA score 29 and had acute retention twice. Cardiologist has advised against discontinuation of anti-
coagulant. Which amongst the following is best way to perform prostatectomy without stopping ant-
coagulant?

a. TURP with Unipolar cautery

b. TURP with bipolar cautery

c. HOLEP

d. KTP laser (Green light laser vaporization

Answer. d

156. Antibiotic prophylaxis is checked in which part of WHO surgical safety check list?

a. Sign in

b. Time in

c. Sign out

d. Time out

Answer. d

157. Lyre sign is seen in?

a. Cystic Hygroma

b. Branchial cyst

c. Thyroglossal cyst

d. Carotid body tumour

Answer. d

158. False statement about Hashimotos thyroiditis is?

a. Common in old males.

b. Anti-TPO antibody is high

c. It is a risk factor for lymphoma.

d. It is a progressive disease so it finally leads hypothyroidism.

Answer. a
159. A 52 year old male presented to your clinic with history of chronic pain and heaviness in legs. O/E
prominent veins are observed in both legs with areas of pigmentation near malleolus. According to
CEAP classification in which group is should be kept?

a. C4a

b. C4b

c. C4c

d. 5

Answer. a
160. Identify the instrument?

a. Ureteric stents

b. ERCP CBD stone extractor balloon

c. PCNL balloon

d. Fogarty embolectomy catheter

Answer. d

161. A 55 yr old man underwent an operation for lipoma. 11 months after surgery he felt severe itching and
burning on the operated site where a scar is formed which is limited to the wound boundary. What is the nature
of this lesion

a. Hypertrophic scar

b. Keloid

c. Subserous lipoma

d. Granulation tissue

Solution. Answer A. hypertrophic Scar

Answer. a
162. A new murmur is identified in the aortic area in a 45-year-old male. He doesn’t have any history
suggestive of rheumatic fever however a few years ago he developed a painless ulcer on penis which
healed on its own after a few weeks. Imaging shows aortic root dilation. Which of the following
pathogenesis is leading to this manifestation?

a. obliterative end arteritis

b. Syphilitic cardiomyopathy

c. Type II hypersensitivity reaction

d. inflammation induced collagen degradation

Solution. ANSWER A: obliterative end arteritis


The patient history is suggestive of a syphilis now presenting with murmurs (? Cardiac syphilis,
there is aortic root dilation suggesting the patient may be suffering from thoracic aneurysm which
is due to obliterative endarteritis of thoracic aortic vasa vasorum.

Answer. a
163. A 42 year female underwent ileectomy, due toa perforated Crohn disease. She now has easy
fatigue, pallor and the tongue examination is as shown below. Her hemoglobin is 9 grams per deciliter.
Which of the following deficiency would have led to such a manifestation?

a. Iron

b. Folic acid

c. Vitamin B6

d. Vitamin B12

Solution. Answer D. vitamin B12. Ileectomy & bald tongue With anemia suggest vitamin B12
deficiency. Vitamin B12 is absorbed from the distal ileum.

Answer. d
164. A drug was studied in mice for its hepatotoxicity. The drug was injected into the mice at a dose of
10mg/kg & 20 mg/kg and biopsy from liver was taken. Hepatocytes appeared increased in size vacuoles
in cytoplasm with intact nuclei at 10 mg/kg dose and there was loss of nuclei increased in size with
vacuoles in the cytoplasm. Which of the following is the correct interpretation of this experiment?

a. A. Drug not toxic at 10 mg/kg but toxic at 20 mg/kg.

b. B. Drug is equally toxic at both the doses.

c. C. Drug is toxic at both doses but toxic at 20 mg/kg.

d. D. Drug is not toxic at both the doses.

Solution. ANSWER. C: Drug is toxic at both doses but toxic at 20 mg/kg. At 10mg the drug is
showing reversible cell injury and at 20mg it is irreversible cell injury thus the answer

Answer. c

165. A 45-year-old male has presented with hemoptysis and hematuria. Physical examination is as
shown below, biopsy from the lung shows noncaseating granulomas. Urine examination shows many
RBC and RBC casts. Serological examination shows elevated c-ANCA. Which of the following is the
most likely diagnosis?

a. Sarcoidosis

b. Granulomatosis with polyangiitis

c. Disseminated tuberculosis

d. Churg Strauss syndrome

Solution. Answer B. granulomatosis with polyangiitis. The clinical image shows saddle nose
along with granulomas in the lung and glomerulonephritis suggests of granulomatosis with
polyangiitis

Answer. b
166. A child born to a Rh-negative mother had a Rh mismatch reaction, Coombs test was positive, what
is the mechanism of this type of reaction (hypersensitivity seen in the child?

a. Anaphylactic

b. Cytotoxic

c. Immune complex mediated

d. Antibody mediated

Solution. Answer D. Antibody mediated. The child is having hemolytic disease of newborn
secondary to RH incompatibility which is type 2 [antibody mediated] hypersensitivity reaction.

Answer. d
167. A 22-year-old soccer player collapses on the field during a practice match. He died on the way to
hospital. The autopsy finding of heart is as shown in the figure. Which of the following would have
prevented death in this patient?

a. Routine daily cardio exercise

b. Diuretics

c. Vasodilators

d. None

Solution. Answer D. None. The image shows the patient died from hypertrophic cardiomyopathy
leading to myocardial infarction. Anything that reduces preload [diuretics, tachycardia and
vasodilators] reduces the cardiac output further, or anything that increases force of contraction
[exercise Inotrope etc] precipitating Mi /heart failure. The medical management off includes beta
blockers [reduce force of contraction and reduce rate of contraction]

Answer. d
168. A 62-year-old man has had anorexia, vomiting, and vague abdominal pain accompanied by weight
loss of 6 kg over the past 2 months. An abdominal CT scan shows the stomach is shrunken with the
gastric wall thickened to 1 cm and with extensive overlying mucosal erosions. Biopsy from lesion in the
stomach is as shown below. Which of the following conditions most likely preceded development of his
illness?

a. Chronic alcoholism

b. Hyperglycemia

c. Pernicious anemia

d. Gastro esophageal reflux

Solution. ANSWER C: Pernicious anemia


This is a case linitis plastica of the stomach , which typically has a signet ring cell pattern of
adenocarcinoma diffusely infiltrating the stomach. Autoimmune gastritis is a risk factor for this
condition. The atropbic gastritis leads to loss of parietal cells that produce intrinsic factor needed
for B12 absorption.

Answer. c
169. A 55-year male is having easy fatigue for last eight months, he has developed excessive itching for
the last four months. Blood examination is as follows Hb 8g/dL, TLC 32000, DLC N-40, L-21, Myelo-6,
metamyelo-6 Band 6, Basophils 6, Monocytes 6, Blasts 9%. Platelet count 1.6L. What is the most likely
diagnosis?

a. Acute myeloid leukemia

b. chronic myeloid leukemia

c. hairy cell leukemia

d. chronic lymphocytic leukemia

Solution. Answer B. chronic myeloid leukemia. The key word is Basophils 6% is suggestive of
CML.

Answer. b
170. A 30-year male presents with swelling in the testis. A high inguinal orchiectomy was performed,
and the gross image is as shown below what is the most likely diagnosis?

a. Seminoma

b. Embryonal carcinoma

c. Metastatic carcinoma

d. Teratoma

Solution. Answer A. Seminoma. homogeneous grey, white tumor without hemorrhage and
necrosis is classically seen in seminomas. The non seminomatous tumor will show large areas of
hemorrhage and necrosis.

Answer. a
171. A 45-year-old male living in a western part of india presents with increasing abdominal pain and jaundice.
He has worked as migrant labour in the farm for many years, and sometimes his grain has become moldy. He
is a known smoker with history of 5-6 cigarettes per day for last 10 years. he consumes moderate amount of
alcohol once or twice a week for last 10 years. Physical examination reveals a large mass involving the right
side of his liver, and a biopsy is as shown in the image. The pathogenesis of this tumor involves which of the
following substances and the possible source?

a. Venyl Chloride- Cigarette smoke


b. Azo Dyes- pesticide
c. Naphthylamine- contaminated ground water

d. Aflatoxin- moulds

Solution. Answer D. histology and history is suggestive of Hepatocellular carcinoma as diagnosis.


Aflatoxin is one of the chemical causing Hepatocellular carcinoma. Different chemicals with their
carcinogenesis is as follows in the table

Answer. d
172. A 44-year-old woman dies as a consequence of a 'stroke'. At autopsy, she is found to have a large
right basal ganglia hemorrhage. She has an enlarged 550 gm heart with predominantly left ventricular
hypertrophy. Her kidneys are as shown in the image. Which of the following is the most likely condition
associated with her findings?

a. Autosomal dominant polycystic kidney disease

b. Autosomal recessive polycystic disease

c. Monkeberge medial sclerosis

d. Hypertensive emergency.

Solution. ANSWER. D: Hypertensive emergency.


The image shows flea-bitten kidney which is characteristically seen in malignant hypertension,
which can lead to cardiac hypertrophy and haemorrhages eventually leading to death.

Answer. d

173. A 15-year boy presents with fever cough and cold for two days, now he has a buot of haematuria.
Urine examination shows RBC and RBC casts. What is the most likely diagnosis?

a. Post streptococcal glomnerulonephritis

b. IgA Nephropathy

c. Henoch Schönlein Purpura

d. Microscopic polyangitis

Solution. Abswer B. IgA Nephropathy. Glomerulonephritis within few days off respiratory tract
infection is suggestive of IGA nephropathy. Option a presents after one to three weeks off
infection option C will have additional purpura arthralgia and abdominal cramps. Option D is seen
in adults is associated with ANCA

Answer. b
174. Supravital stains are used to demonstrate which of the following?

a. Reticulocytes

b. Amyloid

c. Pappenheimer bodies

d. Both A and C

Solution. Answer A. Reticulocyte. Supra vital stains are used for demonstrating reticulocyte and
Heinz bodies.

Answer. a

175. A five-year child is diagnosed off medulloblastoma. Detailed history suggests his father had a
basal cell carcinoma of skin, now you start suspecting a familial cancer syndrome. Which of the
following mutation is most likely to be seen in such an inheritance?

a. PTEN

b. P53

c. PTCH

d. SKT-11

Solution. Answer C. PTCH. medulloblastoma with basal cell carcinoma is characteristically seen
in Gorlin syndrome which is due to PTCH mutation.

Answer. c
176. A 24-year-old female was evaluated for general fatigue. There was no Pallor, Clubbing or oedema
feet on general examination. Auscultation revealed Loud S1 and a mid-diastolic murmur best heard on
expiration. What is the probable diagnosis?

a. Mitral regurgitation

b. Aortic regurgitation

c. Mitral stenosis

d. Aortic stenosis

Solution. Ans 1-C. Mitral Stenosis


Mitral Stenosis is most commonly caused by Rheumatic Heart disease. Since Rheumatic fever
occurs in children and affects the Mitra valve most commonly, MS is diagnosed in Young females.
It is the most common valve defect complicating pregnancy.
MS most commonly presents as Fatigue due to RV strain. Examination reveals Loud S1. Causes of
loud S1- MS, TS, Hyperdynamic states. Short PR interval.
Best heard on expiration indicated Left heart disease. Along with a Mid diastolic murmur, the
most probable diagnosis is MS.
Murmurs in other defects :
Chronic MR : Soft S1, Pan systolic murmur, best heard on expiration, radiating to Axilla/ISA
AS : Ejection systolic murmur on expiration, radiating to the carotids
AR : Early Diastolic murmur.

Answer. c

177. A 60-year-old male patient presented with 6 hours of retrosternal chest pain, constricting in
nature. ECG showed ST segment depression with normal troponin levels repeated twice after 3 hours.
What is the most likely cause of this acute coronary syndrome?

a. Unstable angina

b. Prinzmetal angina

c. Non-ST elevation MI

d. Stable angina

Solution. Ans-A. Unstable Angina


The diagnosis of acute coronary syndrome when patient presented with anginal pain-
1.ST depression + Normal troponin= Unstable angina(classical
2.ST depression + Elevated troponin= Non-ST elevation MI
3.ST elevation(Transient + Normal troponin= Prinzmetal angina
4.ST elevation + Elevated troponin=ST elevation MI

Answer. a
178. Identify the arrythmia in this ECG?

a. Sinus tachycardia

b. Atrial flutter

c. Atrial fibrillation

d. PSVT

Solution. Ans-3. D. PSVT.


Absence of P waves with narrow QRS and regular heart rate is suggestive of PSVT. Approach to
tachyarrhythmia-

Answer. d
179. A 64-year-old Gentleman was admitted with chronic cough and breathlessness. He is a diabetic,
chronic smoker with history of COLD for 5 years on Bronchodilators. He gives history of anorexia and
low-grade fever for a month. There is no edema or angina. On examination, his pulse was 112/min
regular with a BP of 120/74 mm of Hg and RR of 40/min. There was no cyanosis/edema. Clubbing –
Grade 3 was present. What is the most probable diagnosis?

a. COLD exacerbation

b. Cor pulmonale

c. Carcinoma lung

d. Heart failure

Solution. Ans.4. c. Carcinoma lung


COLD is a risk factor for Cor pulmonale and is prone to infective exacerbation. However, COLD
never causes Clubbing. Clubbing along with history of low-grade fever and anorexia favours a
diagnosis of carcinoma lung.

Answer. c

180. Amiodarone is the preferred anti-arrythmic drug in emergency management of tachyarrythmia.


Identify the FALSE statement about Amiodarone

a. Amiodarone is a Potassium channel blocker – Class III agent

b. It has additional Sodium channel blocking property – Class I property

c. It has least risk of Torsade de pointes among Class I & III agents

d. It has best success in reversal of Atrial fibrillation

Solution. Ans.5. d. It has best success in reversal of Atrial fibrillation


Amiodarone is a Potassium channel blocker – Class III agent with additional Sodium channel
blocking property – Class I property. Hence it is most potent to control HR in emergency
management of tachyarrythmia. It also has least risk of Torsade de pointes among Class I & III
agents, making it the most preferred agent in emergency.
However, Ibutilide has best success in reversal of Atrial fibrillation.

Answer. d
181. A HIV positive patient presented with severe odynophagia. Gastroscopy was performed. Identify
the probable diagnosis

a. Esophageal candidiasis

b. CMV esophagitis

c. Herpes Simplex related esophagitis

d. Histoplasma infection

Solution. Ans.6. a. Esophageal candidiasis


Esophageal candidiasis is an AID defining criteria presenting with sever odynophagia with
whitish, flaky lesions on gastroscopy. CMV esophagitis can also be painful but the shallow ulcers
appear reddish.

Answer. a

182. Which of the following site is spared in ulcerative colitis

a. Rectum

b. Sigmoid colon

c. Ascending colon

d. Small intestine

Solution. Ans 7-D. Small intestine


Small Intestine is spared in ulcerative colitis. MC site involved is recto-sigmoid

Answer. d
183. The best marker of hepatitis B virus replication is

a. HbsAg

b. HbeAg

c. HBV-DNA

d. ALT

Solution. Ans-8.C.HBV-DNA is most reliable/sensitive/best/quantitative marker of replication.


HbeAg is qualitative marker of replication but can be negative(inspite of replication in precore
mutants.

Answer. c

184. An 84-year-old female was found to have a blood pressure of 85/60 mmHg, heart rate
101beats/min, temperature 37.8°C. Laboratory data are obtained: sodium 137 meq/L, potassium
2.8meq/L, HCO3 10 meq/L, chloride 117 meq/L, BUN 17 mg/dL, creatinine 0.9 mg/dL. An arterial blood
gas shows PaO2 80 mmHg, PCO2 24 mmHg, pH 7.29. Her urine analysis is clear and has a Ph of 4.5.
What is the acid-base disorder?

a. High Anion-gap metabolic acidosis

b. Normal–anion-gap metabolic acidosis Decompensated

c. Normal–anion-gap metabolic acidosis and compensatory respiratory alkalosis

d. Respiratory acidosis

Solution. Answer.9. c. NAGMA and compensatory respiratory alkalosis.


Steps in ABG:
1. See the pH: Acidosis or Alkalosis
2. Check for primary disorder: Respiratory or metabolic
3. See for compensation.
a.IF primary was respiratory process , calculate metabolic compensation. If primary was
metabolic calculate respiratory compensation
b. If given value is within the expected value then it is a simple acid base disorder
c. If given value is not correlating with expected value then it will be a inappropriate
compensation / mixed disorder.
4. Calculate Anion gap
In given case, Acidosis is correlating more to fall in HCO3 so its metabolic acidosis.
Expected PaCO2 = [1.5 × HCO3] +8 +-2 that give the value of 21 to 25mmhg. Here the given
PaCO2 is correlating with expected value which will suggest metabolic acidosis and compensatory
alkalosis. Anion gap is also with in normal range (10-12meq

Answer. c
185. Aphasia with impaired fluency and naming but with preserved repetition and comprehension-

a. Broca’s aphasia

b. Anterior transcortical

c. Posterior transcortical

d. Conduction aphasia

Solution.

Answer. b
186. A 30-year-old man who has had focal epilepsy for 15 years still experiences four to five episodes
per month despite medication. He takes two anticonvulsants; his current medications are the ninth and
tenth drugs he has tried. Repeated EEGs have shown a right temporal focus. He is on disability and
lives with his parents. What is the next best step in management of this patient?

a. Addition of a third antiepileptic drug to his current regimen

b. Substitution of a new drug for one of his current antiepileptic drugs

c. Surgical evaluation

d. Transcranial magnetic stimulation

Solution.

Answer. c

187. All are true about Sjogren’s syndrome except:

a. Bilateral parotid gland enlargement

b. Xerostomia may present

c. Progression to lymphoma

d. Males are affected more than females

Solution. Ans.12. d. Males are affected more than females


Sicca syndrome is an autoimmune exocrinopathy characterized by lymphocytic infiltration of the
exocrine glands resulting in xerostomia and dry eyes (keratoconjunctivitis sicca.
Middle-aged women (female-to-male ratio, 9:1 are primarily affected. It can be a primary disease
or can co manifest with another auto immune disease ( Secondary sjogren’s like RA.
Most common cause of secondary Sicca is RA.
Enlargement of the parotid or other major salivary glands occurs in two-thirds of patients with
primary Sjögren’s syndrome but is uncommon in those in association with rheumatoid arthritis.
Parotid gland enlargement is usually bilateral and may be painful.

Answer. d
188. Which of the following is not true about the most common ocular manifestations of the following
Rheumatological diseases?

a. Ankylosing Spondylitis – Anterior Uveitis

b. Behcet’s Disease – Panuveitis

c. RA – Keratoconjunctivitis Sicca

d. SLE - Cataract

Solution. Ans 13. D. SLE - Cataract


Most common Ocular feature of SLE is also Keratoconjunctivits Sicca.

Answer. d
189. A 40 year old male presented with 10 days history of Fever , severe joint pains and headache. He
was treated with antibiotics by a doctor previously but he didn’t respond. On examination he was found
to have high BP , features of inflammatory arthritis in large joints , gangrene in 2 fingers of upper limb
and a skin lesion as given in the picture below. Investigations revealed high creatinine without
proteinuria or hematuria. ANA and ANCA were negative. HbsAg turned out +ve but the patient
ultimately succumbed to abdominal perforation.

What is the likely diagnosis ?

a. Henoch Schonlein Purpura

b. Microscopic Polyangitis

c. Polyarteritis Nodosa

d. Fibromuscular Dysplasia

Solution. Not treated with antibiotics probably non infectious


High BP , high creatinine without preoteinuria or hematuria shows Renal artery involvement
without features of glomerulonephritis.
Gangrene and livedo reticularis on skin are characteristic of PAN
negative ANA and ANCA ; with Hep B serology +ve are highly associated .
most common cause of death is abdominal perforation

Answer. c
190. Non selective proteinuria with quantity more than 2g/day is most probably due to:

a. Minimal change disease

b. FSGS

c. Post Streptococcal GN

d. Tubulo-interstitial disorders

Solution. Quantity more than 2g/day is Glomerular range proteinuria.


Nephrotic syndromes (Minimal change disease, FSGS present with selective proteinuria as the major
protein in the urine is Albumin. Nephritic syndromes present with non selective proteinuria.

Answer. c
191. A 10-year-old boy presents with short stature to the clinic for evaluation. The boy appears weak
and complains of bone pains which did no respond to Calcium and D3 supplementation. He has history
of reccurent loin pain and was detected to have nephrocalcinosis on USG and started on Citrate
supplementation 6 months ago. What is the most likely laboratory abnormality in this case?

a. High Anion gap metabolic acidosis

b. Elevated Urinary Chloride level

c. Urinary ph > 5.5

d. Low Urinary Anion gap

Solution. Ans 16. (c Urinary ph > 5.5


The case is of Type 1 RTA. In Type 1 RTA(Distal RTA, there is defect in the H+-K+ ATPase pump.
It the most common inherited RTA. It is the most severe form of RTA presenting in young males as
Bone disease manifesting as Rickets and Short stature.
It is associated with defect in paracellular Calcium and Magnesium leading to Hypercalciuria and
HypoMg++. This associated with Hyocitraturia increases risk of Nephrocalcinosis.
ABG shows Metabolic Acidosis. Inspite of acidemia, the urine pH remains > 5.5 due to severe
defect in acidification defect in urine.
Due to acidification defect, urinary chloride levels are low and Urinary Anion gap is high/positive

Answer. c

192. Which of the following conditions is characteristically associated with hypercalciuria?

a. Liddle syndrome

b. Gitelman syndrome

c. Bartter syndrome

d. Gordon syndrome

Solution.

Answer. c
193. Following findings can be seen in early stages of Diabetic nephropathy EXCEPT

a. Thickening of GBM

b. Supranormal GFR

c. Microalbuminuria

d. Uremic features

Solution. Ans. 18. D. Uremic features occur only in late irreversible stages of the disease.

Answer. d

194. A 62year old Female came with headache and toe pain which relieves with exposure to cold. On
examination there is moderate splenomegaly. CBC revealed a Hb-11gm/dl and Platelet count of
1000*103/microliter. The first line treatment in this patient would be

a. Anagrelide

b. Hydroxyurea

c. Busulphan

d. Ruxolitinib

Solution. Ans.19. b. Hydroxyurea.


Patient is suffering from Essential Thrombocytosis.
Essential thrombocytosis (primary thrombocythemia is a nonreactive, chronic myeloproliferative
disorder in which sustained megakaryocyte proliferation leads to an increase in the number of
circulating platelets.
Most symptomatic patients present with symptoms that relate to small- or large-vessel
thrombosis. Microvascular occlusion of the toes and fingers causes digital pain which increases
with exposure to heat and improves with cold; a single dose of aspirin may provide relief for
several days.
Headache is the most common neurologic symptom.
Cytoreductive therapy should be used to decrease the platelet count in high-risk patients (i.e
those over 60 years of age, those with a history of thrombosis, or platelet counts greater than 1.5
million/μL.
Hydroxyurea is generally considered the first-line drug for cytoreductive therapy in essential
thrombocytosis. Second-line agents include the following:
Busulfan
Anagrelide
Interferon alfa
Ruxolitinib

Answer. b
195. Dupilumab is a Biological for the management of

a. Migraine

b. Refractory Asthma

c. Refractory COLD

d. Usual Interstitial Pneumonia

Solution. Ans.20. b. Asthma prophylaxis


Dupilumab is a monoclonal antibody against IL 4 and down regulated eosinophils and mast cells.

Answer. b

196. Identify the disease associated with the following sign positive.

a. Bulbar Palsy

b. Pseudobulbar palsy

c. Parkinsons disease

d. Syringomyelia

Solution. Ans.21 b. Jaw jerk positive indicates Pseudobulbar palsy


The jaw jerk reflex or the masseter reflex is a stretch reflex used to test the status of a patient's
trigeminal nerve (cranial nerve V and to help distinguish an upper cervical cord compression from
lesions that are above the foramen magnum. The mandible—or lower jaw—is tapped at a
downward angle just below the lips at the chin while the mouth is held slightly open. In response,
the masseter muscles will jerk the mandible upwards. Normally this reflex is absent or very slight.
However, in individuals with upper motor neuron lesions the jaw jerk reflex can be exaggerated.

Answer. b
197. A 22-year-old man developed painful swelling of his left knee and gives a history of diarrhoea 3
weeks back which resolved in 3 days. Testing for gonorrhoea is negative. Reactive arthritis associated
with the recent enteric infection is suspected. Which of the following is true about this condition?

a. Intestinal infection with amoeba has been associated with this syndrome.

b. There is a higher prevalence of HLA-DQ2.

c. It is more common in women than in men.

d. Synovial fluid shows an elevated leukocyte count

Solution. ANSWER 22 D. Post enteric reactive arthritis (also known as Reiter syndrome typically
develops 2–4 weeks after an acute diarrheal illness. It is an immune-mediated synovitis, so the
synovial fluid shows an elevated leukocyte count despite the absence of infection. Gonococcal
arthritis is important to exclude in a young person presenting with acute monoarticular arthritis.
Shigella sp. is the most commonly associated enteric organism associated with this syndrome,
other causes include Salmonella sp., Campylobacter jejuni, Yersinia enterocolitica, and even
Clostridium difficile have been implicated.
Amoebic intestinal infections are not associated with reactive arthritis. People who develop this
syndrome have a higher prevalence of HLA-B27 antigen, not HLADQ2, which is associated with
celiac disease.
This syndrome is more common in men and classically is associated with the triad of arthritis,
conjunctivitis, and urethritis.

Answer. d
198. PAO2-PaO2 gap normal with high PaCO2 is seen in

a. Low FiO2

b. Hypoventilation

c. V:Q mismatch

d. Intracardiac shunting

Solution. Ans.23. B Hypoventilation. (Refer to below approach

Answer. b

199. The severity of COLD is assessed using post bronchodilator spirometry. From the list below,
select the values that you would expect to see in a patient with Moderate COLD:

a. FEV1 >= 80% of predicted

b. FEV1 – 50-79% of predicted

c. FEV1 – 30-49% of predicted

d. FEV1 < 30% of predicted

Solution. B. FEV1 – 50-79% of predicted

Answer. b
200. Following statements about Polycythemia Rubra Vera (PRV are true EXCEPT:

a. It is associated with JAK2 mutation

b. Hyperhomocystinemia maybe a risk for thrombosis

c. Phlebotomy is indicated to lower Hemotocrit to reduce thrombosis risk

d. Blood from patient cannot be transfused to potential donors

Solution. Ans. 25. D. Phlebotomy is indicated in PRV to maintain the hematocrit below 45%. The
blood can be transfused safetly. Hyperhomocystinemia is a risk factor for thrombosis and is also
widely prevalent (35% in controls, 56% in persons with polycythemia vera.

Answer. d
201. 50 year old female who presented to the out patient department with the chief complain of swelling
on the nose associated with difficulty in breathing and nasal obstruction since one and half years. On
examination, thick blood stained foul smelling discharge was seen coming out of the nasal cavity which on
cleaning revealed pinkish grey mass filling the right nasal cavity. The mass bled on manipulation.
Haematological and biochemical investigations were within normal range. Study the given radiological
image correlate the diagnosis:

a. Intranasal melanoma

b. Squamous cell carcinoma

c. Minor salivary tumor

d. Esthesioneuroblastoma

Solution. Ans 1: (d Esthesioneuroblastoma


Ref:Read the text below
Sol

Answer. d
202. A 6 year old Child operated for Adeno- tonsillectomy has developed atlantoaxial subluxation. What is the
possible diagnosis in this case?

a. Kallman’s syndrome

b. Grisel Syndrome

c. Ortner syndrome

d. Eagle syndrome

Solution. Ans 2:- (b Grisel Syndrome


Ref: Read the text below
Sol:

Answer. b

203. What is the purpose of preserving the “keystone area” during septoplasty?

a. To maintain appropriate support of the nasal dorsum to prevent postoperative saddle nose
deformity

b. To keep cartilage available for future rhinoplastic procedures

c. To support the lower lateral cartilages

d. To prevent postoperative epistaxis

Solution. Ans 3: (a To maintain appropriate support of the nasal dorsum to prevent postoperative
saddle nose deformity
Ref:Read the text below
Sol
The perpendicular plate and quadrangular cartilages just underneath the upper lateral cartilages
should be preserved to avoid external nasal deformity

Answer. a
204. 27-year-old otherwise healthy man is admitted to the hospital for a fever of unknown origin. The next
morning he presents with a right-sided jugular vein thrombosis, ptosis, anhidrosis, and miosis. The infection is
most likely located in which deep neck space?

a. Prestyloid parapharyngeal space

b. Post styloid parapharyngeal space

c. Retropharyngeal space

d. Anterior visceral space

Solution. Ans 4: (b Post styloid parapharyngeal space


Ref:Read the text below
Sol

Answer. b

205. The two most common bacterial infections known to cause SSNHL:

a. Lyme disease and syphilis

b. CMV and syphilis

c. Lyme disease and CMV

d. CMV and Measles

Solution. Ans 5: (a Lyme disease and syphilis


Ref:Read the text below
Sol

Answer. a
206. Which of these is the most commonly identified organism in subperiosteal orbital abscess due to
sinusitis?

a. Pseudomonas aeruginosa

b. Haemophilus influenzae

c. Streptococcus pneumoniae

d. E.Coli

Solution. Ans 6: (c Streptococcus pneumoniae


Ref:Read the text below
Sol
Streptococcus pneumoniae is the most common cause of orbital sub- periosteal abscess.
These organisms are also the most common cause of intracranial com- plications.

Answer. c

207. An 28 year old male patient complained of dull aching pain over the forehead for the last 3 years.This
pain increased in the morning and decreased in the afternoons, together with intermittent nasal discharge.
10 days ago the pain became very severe with complete nasal obstruction and fever 38 C the patient did not
receive the proper treatment and by the tenth day became drowsy with some mental behavioral changes, also
there was vomiting and blurred vision.Which of the following is the most common complication associated
with pathology:

a. Mucocele

b. Orbital cellulitis

c. Orbital abscess

d. Superior orbital fissure syndrome

Solution. Ans 7: (a Mucocele


Ref:Read the text below
Sol

Answer. a
208. A child presenting with recurrent respiratory tract infection, mouth breathing and decreased
hearing, study the given toposcopic image and choose the most appropriate statement?

a. Tympanostomy tubes are usually required for treatment


b. Sensorineural deafness occurs as a complication in 80% of the cases

c. Intracranial spread of the infection complicates the clinical courses

d. Gram-positive organisms are grown routinely in culture in the aspirate

Solution.

Surgical treatment of causative factor. Adenoidectomy,tonsillectomy

Answer. a
209. The most common long term complication of Functional endoscopic sinus surgery

a. Atrophic rhinitis

b. Anosmia

c. Intranasal synechiae formation

d. Intermittent nasal bleeds

Solution. Ans 9:- (c Intranasal synechiae formation


Ref: Read the text below
Sol:

Answer. c

210. A 7 year old child while playing fell down. He has developed nasal blockage on both sides and
there is swelling around the nose. What is the further treatment of this child?

a. Aspiration of septal swelling

b. Anterior Nasal packing

c. Start Intravenous steroids

d. Start Topical Corticosteroid nasal spray

Solution. Ans 10:- (a Aspiration of septal swelling


Ref: Read the text below
Sol:
This is a case of traumatic septal hematoma.
The treatment is immediate aspiration or drainage of hematoma else it can lead to septal abscess
and finally to septal perforation

Answer. a

Test Answer

1 (d) 2 (d) 3 (a) 4 (c) 5 (a) 6 (a) 7 (b) 8 (b)


9 (b) 10 (a) 11 (d) 12 (d) 13 (d) 14 (c) 15 (c) 16 (a)
17 (c) 18 (a) 19 (a) 20 (a) 21 (c) 22 (d) 23 (a) 24 (d)
25 (a) 26 (b) 27 (d) 28 (a) 29 (d) 30 (a) 31 (d) 32 (a)
33 (a) 34 (b) 35 (d) 36 (c) 37 (c) 38 (d) 39 (b) 40 (c)
41 (d) 42 (b) 43 (c) 44 (b) 45 (d) 46 (a) 47 (a) 48 (b)
49 (a) 50 (b) 51 (d) 52 (b) 53 (c) 54 (b) 55 (c) 56 (b)
57 (c) 58 (a) 59 (a) 60 (a) 61 (a) 62 (c) 63 (c) 64 (d)
65 (c) 66 (b) 67 (c) 68 (b) 69 (c) 70 (d) 71 (d) 72 (b)
73 (b) 74 (c) 75 (b) 76 (d) 77 (d) 78 (a) 79 (c) 80 (b)
81 (b) 82 (d) 83 (b) 84 (b) 85 (a) 86 (c) 87 (c) 88 (b)
89 (b) 90 (b) 91 (b) 92 (a) 93 (b) 94 (b) 95 (b) 96 (c)
97 (c) 98 (c) 99 (b) 100 (a) 101 (c) 102 (b) 103 (a) 104 (a)
105 (c) 106 (a) 107 (c) 108 (d) 109 (b) 110 (b) 111 (d) 112 (a)
113 (b) 114 (c) 115 (a) 116 (c) 117 (d) 118 (b) 119 (c) 120 (b)
121 (d) 122 (a) 123 (c) 124 (a) 125 (c) 126 (d) 127 (c) 128 (c)
129 (a) 130 (c) 131 (a) 132 (a) 133 (b) 134 (a) 135 (c) 136 (c)
137 (a) 138 (b) 139 (d) 140 (d) 141 (d) 142 (b) 143 (b) 144 (b)
145 (a) 146 (a) 147 (a) 148 (a) 149 (d) 150 (b) 151 (d) 152 (a)
153 (a) 154 (b) 155 (d) 156 (d) 157 (d) 158 (a) 159 (a) 160 (d)
161 (a) 162 (a) 163 (d) 164 (c) 165 (b) 166 (d) 167 (d) 168 (c)
169 (b) 170 (a) 171 (d) 172 (d) 173 (b) 174 (a) 175 (c) 176 (c)
177 (a) 178 (d) 179 (c) 180 (d) 181 (a) 182 (d) 183 (c) 184 (c)
185 (b) 186 (c) 187 (d) 188 (d) 189 (c) 190 (c) 191 (c) 192 (c)
193 (d) 194 (b) 195 (b) 196 (b) 197 (d) 198 (b) 199 (b) 200 (d)
201 (d) 202 (b) 203 (a) 204 (b) 205 (a) 206 (c) 207 (a) 208 (a)
209 (c) 210 (a)

You might also like